SlideShare a Scribd company logo
ĐẠI HỌC QUỐC GIA HÀ NỘI
TRƯỜNG ĐẠI HỌC KHOA HỌC TỰ NHIÊN
TRẦN THỊ VIẾT THỦY
MỘT SỐ DẠNG TOÁN
VỀ ĐA THỨC QUA CÁC
KỲ THI OLYMPIC
LUẬN VĂN THẠC SỸ TOÁN HỌC
HÀ NỘI - NĂM 2017
ĐẠI HỌC QUỐC GIA HÀ NỘI
TRƯỜNG ĐẠI HỌC KHOA HỌC TỰ NHIÊN
TRẦN THỊ VIẾT THỦY
MỘT SỐ DẠNG TOÁN
VỀ ĐA THỨC QUA CÁC
KỲ THI OLYMPIC
LUẬN VĂN THẠC SỸ TOÁN HỌC
Mã số: 60.46.01.13
Người hướng dẫn khoa học
GS. TSKH. NGUYỄN VĂN MẬU
HÀ NỘI - NĂM 2017
Mục lục
Mở đầu 1
1 Xác định đa thức 3
1.1 Một số tính chất cơ bản của đa thức . . . . . . . . . . . . . 3
1.2 Xác định đa thức theo các đặc trưng số học . . . . . . . . . 5
1.3 Xác định đa thức theo các đặc trưng nghiệm . . . . . . . . 13
1.4 Xác dịnh đa thức theo phép biến đổi vi phân hàm . . . . . 19
2 Ước lượng đa thức 28
2.1 Đa thức Chebyshev và các tính chất . . . . . . . . . . . . . 28
2.2 Các dạng toán liên quan đến đa thức Chebyshev . . . . . . 32
2.3 Ước lượng, giá trị cực trị của đa thức . . . . . . . . . . . . 36
3 Một số dạng toán liên quan 47
3.1 Đa thức với hệ số nguyên và đa thức nhận giá trị nguyên . . 47
3.2 Đa thức với hệ số hữu tỷ và phân thức hữu tỷ . . . . . . . . 58
3.3 Ứng dụng tính chất nghiệm của đa thức . . . . . . . . . . . 67
Kết luận 72
Tài liệu tham khảo 74
1
MỞ ĐẦU
Một chuyên đề cơ bản và quan trọng trong đại số, trong toán học nói
chung là chuyên đề đa thức. Đa thức có vị trí quan trọng trong kiến thức
toán nói chung, trong chương trình phổ thông, và đặc biệt đối với các lớp
chuyên toán nói riêng. Trong các kì thi chọn học sinh giỏi toán, vô địch
Quốc gia, Quốc tế và Olympic sinh viên, các dạng toán về đa thức thường
xuất hiện với mức độ khó và rất khó. Nhiều đề thi cùng đáp án đã được
đăngtải ở tạp chí toán học và tuổi trẻ, ở nhiều sách tham khảo nhưng chưa
thật đầy đủ. Với mong muốn có một chuyên đề giúp nâng cao kiến thức
về đa thức và bồi dưỡng học sinh giỏi toán, luận văn "Một số dạng toán
về đa thức qua các đề thi Olympic” nhằm tìm hiểu, thu thập các tài liệu
biên soạn gồm các đề thi học sinh giỏi toán THPT Quốc gia, đề thi toán
Quốc tế, đề thi Olympic sinh viên.
Các dạng toán về đa thức rất phong phú, đa dạng về thể loại và
phương pháp, thường rất rất phức tạp nên khó phân loại và hệ thống
thành các chuyên đề riêng biệt .Tuy vậy, để đáp ứng nhu cầu về giảng dạy,
học tập, luận văn "Một số dạng toán về đa thức qua các đề thi Olympic”
cũng cố gắng tối đa sắp xếp theo trình tự hợp lí nhằm giúp tiếp cận từng
bước , từng mức độ kiến thức và luyện tập kĩ năng giải toán.
Luận văn được chia làm 3 chương.
Chương 1. Xác định và tồn tại đa thức.
Chương 2. Ước lượng đa thức
Chương 3. Một số dạng toán liên quan đến đa thức
Để hoàn thành luận văn này, tác giả xin được gửi lời cảm ơn sâu sắc
tới GS.TSKH Nguyễn Văn Mậu đã dành thời gian hướng dẫn, chỉ bảo tận
tình, giúp đỡ trong suốt quá trình xây dựng đề cương cũng như hoàn thành
luận văn. Tác giả xin gửi lời cảm ơn chân thành tới các quý thầy cô đã
1
đọc, kiểm tra, đánh giá và đưa ra những ý kiến quý báu để luận văn được
đầy đủ và phong phú hơn.Qua đây, tác giả xin cảm ơn Ban Giám Hiệu,
phòng sau Đại học, khoa Toán Tin trường Đại Học Khoa học Tự Nhiên
Hà Nội đã giảng dạy, tạo điều kiện thuận lợi trong suốt quá trình học tập.
Tuy bản thân đã có nhiều cố gắng, nỗ lực nghiên cứu, song do điều
kiện và trình độ còn hạn chế nên luận văn khó tránh khỏi những sai sót.
Tác giả kính mong nhận được sự đóng góp ý kiến của các thầy cô để bản
luận văn được hoàn thiện hơn! Tác giả xin chân thành cảm ơn!
Hà Nội, tháng 10 năm 2016
Tác giả
Trần Thị Viết Thủy
2
Chương 1
Xác định đa thức
1.1 Một số tính chất cơ bản của đa thức
Định nghĩa 1.1 (xem [2]). Cho vành A là một vành giao hoán có đơn vị.
Ta gọi đa thức bậc n biến x là một biểu thức có dạng
Pn(x) = anxn
+ an−1xn−1
+ · · · + a1x + a0(an 6= 0),
trong đó các ai ∈ A được gọi là hệ số, an là hệ số bậc cao nhất và a0 là hệ
số tự do của đa thức.
Nếu ai = 0, i = 0, · · · , n − 1 và a0 6= 0 thì ta có bậc của đa thức là 0.
Nếu ai = 0 ∀i = 0, · · · , n − 1 thì ta coi bậc của đa thức là −∞ và
gọi là đa thức không.
Tập hợp tất cả các đa thức với hệ số lấy trong vành A được kí hiệu
là A[x].
Khi A = K là một trường thì vành K[x] là một vành giao hoán có
đơn vị. Ta thường xét A = Z,hoặc A = Q hoặc A = R. Khi đó ta có các
vành đa thức tương ứng là Z[x], Q[x], R[x].
Tính chất 1.1 (xem [2]). Nếu các đa thức f(x) và g(x) nguyên tố cùng
nhau và các đa thức f(x) và h(x) nguyên tố cùng nhau thì các đa thức
f(x) và g(x)h(x) cũng nguyên tố cùng nhau.
Tính chất 1.2 (xem [2]). Nếu các đa thức f(x), g(x), h(x) thỏa mãn điều
kiện f(x)h(x) chia hết cho g(x), g(x) và h(x) nguyên tố cùng nhau thì
f(x) chia hết cho g(x).
3
Tính chất 1.3 (xem [2]). Nếu đa thức f(x) chia hết cho các đa thức g(x)
và h(x) với g(x) nguyên tố cùng nhau thì f(x) chia hết cho g(x)h(x).
Tính chất 1.4 (xem [2]). Nếu các đa thức f(x) và g(x) nguyên tố cùng
nhau thì [f(x)]m
và [g(x)]n
cũng nguyên tố cùng nhau với mọi m, n nguyên
dương.
Định lý 1.1 (xem [7]). [Định lí về nghiệm của đa thức]
Nếu một đa thức bậc n và có hệ số của số hạng có bậc cao nhất khác
0 thì nó có không quá n nghiệm.
Định lý 1.2 (xem [7]). [Định lí Bezout]
Cho đa thức P(x) ∈ R[x] và số thực α, khi đó α là nghiệm của
P(x) khi và chỉ khi P(x)
.
.
.(x − α). Điều này có nghĩa là tồn tại đa thức
Q(x) ∈ R[x] sao cho P(x) = (x − α).Q(x).
Định lý 1.3 (Công thức khai triển Abel). Cho bộ số đôi một khác nhau
x1, x2, . . . , xn. Khi đó mọi đa thức P(x) với degP(x) < n+1 đều viết được
dưới dạng
P(x) = a0 + a1 (x − x1) + a2 (x − x1) (x − x2)
+ · · · + an (x − x1) (x − x2) . . . (x − xn) .
Định lý 1.4 (Định lí Viet thuận). Cho đa thức
P(x) = a0xn
+ a1xn−1
+ a2xn−2
+ . . . + an(a0 6= 0)
có n nghiệm là x1, x2, x3, . . . , xn. Khi đó ta có















S1 = x1 + x2 + . . . + xn = −
a1
a0
S2 = x1x2 + x1x3 + . . . + x1xn + . . . + xn−1xn =
a2
a0
. . . . . . . . . . . . . . . . . . . . . . . . . . . . . . . . . . . .
Sn = x1x2 . . . xn = (−1)n an
a0
.
Định lý 1.5 (Định lí Viet đảo). Ngược lại nếu có các số x1, x2, x3, . . . , xn
thỏa mãn







x1 + x2 + . . . + xn = S1
x1x2 + x1x3 + . . . + x1xn + . . . + xn−1xn = S2
. . . . . . . . . . . . . . . . . . . . . . . . . . . . . . . . . . . .
x1x2 . . . xn = Sn.
4
thì x1, x2, x3, . . . , xn là các nghiệm của đa thức
P(x) = xn
− S1xn−1
+ S2xn−2
+ . . . + (−1)n
Sn.
Định lý 1.6 (Định lí Lagrange). Nếu f(x) là hàm liên tục trên đoạn [a, b],
khả vi trên khoảng (a, b) thì tồn tại c ∈ (a, b) sao cho
f0
(c) =
f(b) − f(a)
b − a
.
Một hệ quả rất quan trọng, được áp dụng nhiều trong giải toán của
định lí Lagrange, đó là định lí Rolle:
Định lý 1.7 (Định lí Rolle ). Cho f(x) là hàm liên tục trên đoạn [a, b],
khả vi trên khoảng (a, b) và f(a) = f(b) thì tồn tại c ∈ (a, b) sao cho
f0
(c) = 0.
Định lý 1.8 (Bất đẳng thức Schur ). Cho các số không âm a, b, c. Khi đó
với mọi r > 0 ta có bất đẳng thức
ar
(a − b)(a − c) + br
(b − c)(b − a) + cr
(c − a)(c − b) ≥ 0.
Đẳng thức xảy ra khi và chỉ khi a = b = c hoặc a = b, c = 0 và các hoán
vị tương ứng.
Các trường hợp thường được dùng để giải toán là r = 1, r = 2.
1.2 Xác định đa thức theo các đặc trưng số học
Trong phần này ta khảo sát các bài toán về xác định đa thức với hệ
số nguyên và đa thức nhận giá trị nguyên trên tập số tự nhiên dựa vào các
đặc trưng số học như: tính chia hết, đồng dư, nguyên tố cùng nhau, ...
Bài toán 1.1 (Mathemmatical Reflection issue 4, 2015). Tìm tất cả các
đa thức P(x) bậc ≥ 1 với hệ số nguyên và thỏa mãn điều kiện
a2
+ b2
− c2
| P(a) + P(b) − P(c), ∀a, b, c ∈ Z, (a2
+ b2
− c2
6= 0).
Lời giải. Ta có
a2
+ b2
− c2
| P(a) + P(b) − P(c), ∀a, b, c ∈ Z. (1.1)
5
Chọn b = c, trong (1.1) ta có
P(a)
.
.
.a2
, ∀a ∈ Z.
Suy ra
P(a) = ma2
, ∀a ∈ Z, m ∈ Z. (1.2)
Chọn b = 0, trong (1.1) ta được
a2
− c2
| P(a) + P(0) − P(c), ∀a, b, c ∈ Z. (1.3)
Theo định lý về phương trình Pythagoras, luôn tồn tại vô số các cặp số
nguyên (a, b) sao cho a2
+ b2
= m2
, m ∈ Z. Gọi tập hợp gồm các cặp số
nguyên (a, b) như thế là S. Theo (1.3) ta có
a2
+ b2
− c2
| P(
√
a2 + b2) + P(0) − P(c), ∀a, b ∈ S, c ∈ Z. (1.4)
Từ (1.1) và (1.4), ta suy ra
a2
+ b2
− c2
| P(
√
a2 + b2) + P(0) − P(a) − P(b), ∀a, b ∈ S, c ∈ Z.
Hay
a2
+ b2
− c2
| P(
√
a2 + b2) − P(a) − P(b), ∀a, b ∈ S, c ∈ Z do (1.2).
Cho c → +∞ ta thu được P(
√
a2 + b2) = P(a) + P(b), ∀a, b ∈ S.
Vậy P(
√
a2 + b2) = P(a) + P(b).
Chọn a = b = x ta được P(x
√
2) = 2P(x), ∀x ∈ R.
Giả sử rằng P(x) = anxn
+an−1xn−1
+. . .+a1x+a0, ai ∈ Z, ∀i = 0, n,
sau đó so sánh hệ số bậc cao nhất tương ứng ở hai vế ta được
an(
√
2)n
= 2an ⇒ n = 2.
Suy ra P(x) = a2x2
+ a1x + a0, do P(0) = 0 nên a0 = 0.
Lại từ P(x)
.
.
.x2
, ∀x ∈ Z nên a1 = 0.
Vậy đa thức cần tìm là P(x) = kx2
, k ∈ Z tùy ý khác 0.
Bài toán 1.2 (Olympic SV, 1996). Cho Pn(x) là đa thức bậc n và cho
m ∈ N∗
. Chứng minh rằng
6
Nếu Pn(xm
) chia hết cho (x − a)k
thì nó chia hết cho (xm
− am
)k
(a 6= 0).
Lời giải.
Giả sử,
Pn(x) = an(x − am
)n
+ · · · + a2(x − am
)2
+ a1(x − am
) + a0.
Khi đó
Pn(xm
) = an(xm
− am
)n
+ · · · + a2(xm
− am
)2
+ a1(xm
− am
) + a0.
Ta chứng minh
a0 = a1 = · · · = ak−1 = 0
bằng phương pháp phản chứng.
Thật vậy, giả sử ai là số khác không đầu tiên, trong đó 0 6 ai 6 k−1.
Dễ thấy rằng Pn(xm
) không chia hết cho (x − a)i + 1, với i + 1 > k.
Suy ra Pn(xm
) không chia hết cho (x − a)k
, mâu thuẫn.
Suy ra điều phải chứng minh.
Đặc biệt khi k = a = 1, ta có Pn(xm
) chia hết cho x − 1 thì nó chia
hết cho xm
− 1.
Bài toán 1.3 (Olympic SV, 2002). Tồn tại hay không tồn tại một đa thức
P(x) bậc 2002 sao cho P(x2
− 2001) chia hết cho P(x)?
Lời giải. Ta giả sử tồn tại đa thức P(x) với deg P(x) = 2002.
Xét đa thức P(x) = (x + a)2002
. Ta có
P(x2
− 2001) = (x2
− 2001 + a)2002
=

(x + a)2
− 2a(x + a) + a2
+ a − 2001
2002
.
Nếu ta chọn được a, sao cho a2
+ a − 2001 = 0 hay a =
−1 +
√
8005
2
hoặc a =
−1 −
√
8005
2
, thì đa thức P(x2
− 2001) = (x2
− a2
)2002
=
(x + a)2002
(x − a)2002
chia hết cho P(x). Vậy, đa thức
P(x) =

x +
−1 +
√
8005
2
2002
7
hoặc đa thức
P(x) =

x +
−1 −
√
8005
2
2002
thoả mãn điều kiện bài toán.
Lời bàn: Vì sao lại xét đa thức P(x) = (x + a)2002
như vây? Ta xét
từ bài toán đơn giản trước Tồn tại hay không tồn tại một đa thức P(x)
bậc 2 sao cho P(x2
− 1) chia hết cho P(x)? 
Xét đa thức P(x) = (x + a)2
và chỉ ra được tồn tại đa thức, do đó
bài toán trên xét P(x) = (x + a)2002
.
Từ đó có thể nâng bài toán với bậc của đa thức P(x) cao hơn :
Tồn tại hay không tồn tại một đa thức P(x) bậc 2018 sao cho P(x2
−
2017) chia hết cho P(x)? 
Phát triển thành bài toán tổng quát hơn:
Có tồn tại hay không tồn tại một đa thức P(x) bậc k sao cho P(x2
−k)
chia hết cho P(x) ( k là số nguyên dương.)
Bài toán 1.4 (HSGQG, 2015). Cho fn(x) là dãy đa thức xác định bởi
f0(x) = 2, f1(x) = 3x, . . . , fn(x) = 3xfn−1(x)+(1−x−2x2
)fn−2(x) ∀n ≥ 2.
Tìm tất cả các số nguyên dương n để fn(x)
.
.
. (x3
− x2
+ x).
Lời giải. Từ công thức truy hồi ta có
fn(x) = 3x · fn−1(x) − (x + 1)(2x − 1)fn−2(x)
= (x + 1 + 2x − 1)fn−1(x) − (x + 1)(2x − 1)fn−2(x)
= (x + 1)fn−1(x) + (2x − 1)[fn−1(x) − (x + 1)fn−2(x)].
Nên
fn(x) − (x + 1)fn−1(x) = (2x − 1)[fn−1(x) − (x + 1)fn−2(x)]
fn−1(x) − (x + 1)fn−2(x) = (2x − 1)[fn−2(x) − (x + 1)fn−3(x)]
fn−2(x) − (x + 1)fn−3(x) = (2x − 1)[fn−3(x) − (x + 1)fn−4(x)]
. . . . . . . . . . . . . . . . . . . . . . . . . . . . . . . . . . . .
f2(x) − (x + 1)f1(x) = (2x − 1)[f1(x) − (x + 1)f0(x)].
8
Suy ra
fn(x) − (x + 1)fn−1(x) = (2x − 1)n−1
[f1(x) − (x + 1)f0(x)]
= (2x − 1)n−1
(x − 2).
Do đó
fn(x) − (2x − 1)n
= (x + 1)fn−1(x) + (2x − 1)n−1
(x − 2) − (2x − 1)n
.
Vì vậy,
fn(x) − (2x − 1)n
= (x + 1)fn−1(x) + (2x − 1)n−1
(x − 2 − 2x + 1)
= (x + 1)[fn−1(x) − (2x − 1)n−1
]
= (x + 1)n
[f0(x) − (2x − 1)0
] = (x + 1)n
.
Vậy fn(x) = (2x−1)n
+(x+1)n
. Đặt Q(x) = x3
−x2
+x = x(x2
−x+1).
Vì fn(x)
.
.
. Q(x) nên fn(0) = 0 hay 1n
+ (−1)n
= 0 nên n lẻ.
fn(−2) = (−5)n
+ (−1)n
= −(5n
+ 1) (do n lẻ) chia hết cho
Q(−2) = −2[(−2)2
− (−2) + 1] = (−2)7. Vì fn(−2) = −(5n
+ 1) lại
là số chẵn nên fn(−2)
.
.
. 7. Do 125 ≡ −1 (mod 7) nên xét các trường hợp
sau
• n = 3k, k lẻ ta có 5n
+ 1 = 53k
+ 1 ≡ (−1)k
+ 1 ≡ 0 (mod 7),
• n = 3k + 1, k chẵn ta có 5n
+ 1 = 5 · 53k
+ 1 ≡ 6 (mod 7),
• n = 3k + 2, k lẻ ta có 5n
+ 1 = 25 · 53k
+ 1 ≡ −24 ≡ 3 (mod 7).
Suy ra điều kiện cần của n là n = 3k với k lẻ. Khi đó fn(x) = (x + 1)3k
+
(2x − 1)3k
chia hết cho (x + 1)3
+ (2x − 1)3
, ∀k.
Nhận thấy (x + 1)3
+ (2x − 1)3
= (9x3
− 9x2
+ 9x)
.
.
. g(x) nên ta có
x = 3k với k là số tự nhiên lẻ thỏa mãn, đặt k = 2m + 1 với m nguyên
dương thì n = 6n + 3.
Vậy tất cả các số n cần tìm dạng 6m + 3 với m là số nguyên dương.
Nhận xét: Ta có thể tìm ra được công thức tổng quát của dãy đa
thức đã cho bằng cách coi x là hằng số và xây dựng số hạng của dãy số
9
tương ứng, tức là xét dãy số

u0 = 2, u1 = 3x,
un = 3xun−1 + (1 − x − x2
)un−2
với x là tham số thực nào đó.
Xét phương trình đặc trưng t2
− 3xt + 2x2
+ x − 1 = 0, suy ra
t = x + 1, t = 2x − 1.
Do đó un = α(x + 1)n
+ β(2x − 1)n
.Dựa vào u0, u1 ta tìm được
α = β = 1.
Do vậy, fn(x) = (2x − 1)n
+ (x + 1)n
.
Bài toán thuộc dạng về tính chia hết của đa thức kết hợp với đa thức
xác định bởi hệ thức truy hồi.
Dưới đây là một bài tương tự: Cho fn(x) là dãy đa thức xác định bởi
công thức

f0(x) = 2, f1(x) = 2x + 2,
fn+2(x) = (2x + 2)fn+1(x) − (x2
+ 2x − 3)fn(x), n ≥ 1
Tìm tất cả các giá trị của n sao cho fn(x) chia hết cho x2
+ 2x + 5.
Bài toán 1.5 (IMO Shortlisted 2002). Cho m, n ∈ N(m, n  2) và các
số nguyên a1, a2, ..., an sao cho không có số nào trong chúng chia hết cho
mn−1
. Chứng minh rằng tồn tại các số nguyên e1, e2, ..., en không đồng
thời bằng 0 sao cho |ei|  m, ∀i và e1a1 + e2a2 + · · · + enan chia hết cho
mn
.
Lời giải. Giả sử không tồn tại các số nguyên e1, e2, ..., en không đồng thời
bằng 0 sao cho |ei|  m, ∀i và e1a1 + e2a2 + · · · + enan chia hết cho mn
.
Xét tập A gồm các số có dạng
n
P
i=1
eiai, với 0 ≤ ei ≤ m − 1, i = 1, n.
Trong A có tất cả mn
số và do giả thiết phản chứng ta suy ra mn
số
này lập thành hệ thặng dư đầy đủ (mod mn
) (vì nếu không như vậy thì
có hai số cùng số dư khi chia cho mn
và hiệu hai số này thỏa mãn đề bài.
Vô lý).
Xét f(x) =
P
a∈A
xa
, ta thấy
f(x) =
n
Y
i=1
 m−1
X
j=0
xjai

=
n
Y
i=1
1 − xmai
1 − xai
. (1.5)
10
Xét số phức ε = cos
 2π
mn

+ i sin
 2π
mn

. Do mn
phần tử của A lập
thành hệ thặng dư đầy đủ (mod mn
) nên ta phải có f(ε) = 0. Do đó từ
(1.5)ta suy ra
n
Y
i=1
(1 − εmai
) = 0.
Tuy nhiên điều này mâu thuẫn với giả thiết các số a1, a2, ..., an là các
số nguyên sao cho không có số nào trong chúng chia hết cho mn−1
. Điều
đó chứng tỏ phản chứng là sai. Vậy bài toán được chứng minh.
Bài toán 1.6 (Olympic SV, 2004). Xác định đa thức
f(x) = x5
− 3x4
+ 2x3
+ ax2
+ bx + c biết rằng nó chia hết cho đa
thức (x − 1)(x + 1)(x − 2).
Lời giải. Ta có f(x) chia hết cho (x − 1)(x + 1)(x − 2) khi và chỉ khi
f(1) = a + b + c = 0,
f(−1) = a − b + c − 6 = 0,
f(2) = 4a + 2b + c = 0.
Giải hệ này ta thu được a = 1, b = −3, c = 2.
Vậy đa thức cần tìm là f(x) = x5
− 3x4
+ 2x3
+ x2
− 3x + 2.
Bài toán 1.7. Tìm tất cả các đa thức P(x) bậc 5 thỏa mãn các điều kiện
sau:
Đa thức (P(x) + 1) chia hết cho (x − 1)3
và đa thức (P(x) − 1) chia
hết cho (x + 1)3
.
Lời giải. Từ giả thiết suy ra deg P0
(x) = 4 và P0
(x) chia hết cho (x−1)2
và (x + 1)2
.
Vậy nên
P0
(x) = a(x − 1)2
(x + 1)2
= a(x4
− 2x2
+ 1)
và
P(x) = a
x5
5
− 2
x3
3
+ x + b

.
11
Kết hợp với điều kiện P(1) = −1 và P(−1) = 1 ta thu được
P(x) = −
1
8
(3x5
− 10x3
+ 15).
Thử lại ta thấy nghiệm này không thỏa mãn. Vậy không tồn tại đa
thưc bậc 5 thỏa mãn điều kiện bài ra.
Bài toán 1.8. Tìm đa thức bậc 3 dạng
f(x) = x3
+ ax2
+ bx + c
sao cho f(x) chia hết cho (x − 2) và f(x) chia cho x2
− 1 thì dư 2x.
Lời giải. Vì f(x) chia hết cho x − 2 nên
f(2) = 8 + 4a + 2b + c = 0.
Do f(x) chia cho x2
− 1 thì dư 2x nên g(x) = f(x) − 2x chia hết cho
x2
− 1.
Suy ra
g(1) = 1 + a + (b − 2) + c = 0
hay
a + b + c = 1
và
g(−1) = −1 + a − b + 2 + c = 0
hay a − b + c = −1.
Từ đó ta nhận được a = −10, c = −10, b = −19.
Vậy đa thức cần tìm có dạng
f(x) = x3
− 10x2
− 19x − 10.
Bài toán 1.9. Xác định đa thức bậc n dạng
f(x) = xn
+ an−1xn−1
+ · · · + a1x + a0
biết rằng khi chia f(x) cho (x − b1), (x − b2), · · · , (x − bn)(bi ∈ Z, bi 6=
bj nếu i 6= j)đều có chung số dư là m(m ∈ Z).
12
Lời giải. Từ giả thiết suy ra f(bi) = m(i = 1, 2, · · · , n).
Đặt f(x) − m = g(x) thì deg g = n và hệ số cao nhất của g(x) bằng
1 và g(x) có n nghiệm phân biệt là b1, b2, · · · , bn.
Xét đa thức
h(x) = g(x) − (x − b1)(x − b2) · · · (x − bn).
Khi đó
h(x) = (an−1 − A1)xn−1
+ (an−2 − A2)xn−2
+ · · · + (a0 − An−m),
trong đó A1, A2, · · · , An được xác định như sau







A1 = (−1)1
(b1 + b2 + · · · + bn)
A2 = (−1)2
(b1b2 + b1b3 + · · · + bn−1bn)
. . . . . . . . . . . . . . . . . . . . . . . . . . . . . . . . . . . .
An = b1 + b2 + · · · + bn.
1.3 Xác định đa thức theo các đặc trưng nghiệm
Bài toán 1.10 (Kì thi chọn đội tuyển HSG TPHCM, 2012 - 2013). Tìm
tất cả các đa thức P(x) hệ số thực thỏa mãn điều kiện
P(x) · P(x − 3) = P(x2
), ∀x ∈ R. (1.6)
Lời giải. TH1: P(x) ≡ C (C là hằng số thực) thỏa mãn (1.6). Suy ra
c2
= c nên c = 0 hoặc c = 1. Do đó P(x) = 0 hoặc P(x) = 1.
TH2: deg P(x) ≥ 1.
Gọi α là một nghiệm phức tùy ý của P(x). Từ (1.6) thay x = α
ta có P(α2
) = 0, suy ra x = α2
cũng là một nghiệm của P(x). Từ đó
có α, α2
, α4
, . . . , α2n
cũng các nghiệm của P(x) mà P(x) chỉ có hữu hạn
nghiệm (do đang xét P(x) khác đa thức không), suy ra
h
|α| = 0
|α| = 1. (I)
Từ (1.6) lại thay x = α + 3, ta có P((α + 3)2
) = 0, suy ra x = (α + 3)2
là nghiệm của P(x). Từ x = (α + 3)2
là nghiệm của P(x), tương tự phần
13
trên có (α + 3)2
, (α + 3)4
, (α + 3)8
, (α + 3)16
, . . . là các nghiệm của P(x)
mà P(x) chỉ có hữu hạn nghiệm, suy ra
h
|(α + 3)|2
= 0
|α + 3|2
= 1.
Hay h
|(α + 3)| = 0
|α + 3| = 1. (II)
Như vậy, nếu α là nghiệm của P(x) thì ta có α thỏa mãn hệ (I) và (II).
Từ biểu diễn số phức α thỏa mãn (I) và (II) trên mặt phẳng phức ta
thấy hệ trên không có nghiệm. Suy ra không tồn tại đa thức hệ số thực
P(x) bậc lớn hơn hoặc bằng 1 thỏa mãn (1.6).
Kết luận. Các đa thức P(x) hệ số thực thỏa mãn (1.6) là P(x) = 0
hoặc P(x) = 1.
Bài toán 1.11 (Moldova MO 2004). Tìm đa thức P(x) hệ số thực thỏa
mãn điều kiện (x3
+3x2
+3x+2)P(x−1) = (x3
−3x2
+3x−2)P(x), ∀x ∈ R.
Lời giải. Trước hết ta tìm nghiệm của P(x). Từ giả thiết, ta có
(x + 2)(x2
+ x + 1)P(x − 1) = (x − 2)(x2
− x + 1)P(x). (1.7)
Từ đây, ta chọn x = −2 sẽ được P(−2) = 0. Chọn x = −1 được P(−1) =
0, chọn x = 0 được P(0) = 0 và khi x = 1 thì được P(1) = 0. Suy ra
P(x) = x(x − 1)(x + 1)(x + 2)Q(x), với Q(x) là đa thức hệ số thực. Thay
P(x) vào (1.7), ta được
(x2
+ x + 1)Q(x − 1) = (x2
− x + 1)Q(x), ∀x 6= {0, ±1, ±2}
⇔
Q(x − 1)
x2 − x + 1
=
Q(x)
(x2 + x + 1)
, ∀x 6= {0, ±1, ±2}
⇔
Q(x − 1)
(x − 1)2 + (x − 1) + 1
=
Q(x)
(x2 + x + 1)
, ∀x 6= {0, ±1, ±2}.
Đặt R(x) =
Q(x)
(x2 + x + 1)
, ta có R(x) = R(x−1) với mọi ∀x 6= {0, ±1, ±2}.
Suy ra R(x) ≡ C hằng số nên Q(x) = C(x2
+ x + 1).
Do đó P(x) = Cx(x − 1)(x + 1)(x + 2). Thử lại ta thấy thỏa mãn.
Vậy P(x) = Cx(x − 1)(x + 1)(x + 2).
14
Bài toán 1.12 (Bulgary MO, 2004). Tìm tất cả các cặp đa thức P(x), Q(x)
thuộcR[x] bậc ≥ 1 và thỏa mãn điều kiện
P(x)Q(x + 1) ≡ P(x + 2017)Q(x).
Lời giải. Đặt R(x) = P(x)P(x + 1) · · · P(x + 2016), ta có
P(x) · Q(x + 1) ≡ P(x + 2017)Q(x)
Suy ra
Q(x)
Q(x + 1)
=
P(x)
P(x + 2017)
=
P(x)P(x + 1) · · · P(x + 2016))
P(x + 1)P(x + 2) · · · P(x + 2017)
=
R(x)
R(x + 1)
.
Nếu x lớn hơn nghiệm lớn nhất của P(x) thì
Q(x)
R(x)
=
Q(x + 1)
R(x + 1)
.
Suy ra, với mọi số tự nhiên n, ta có
Q(x)
R(x)
=
Q(x + n)
R(x + n)
.
Cho n → +∞ thì
Q(x + n)
R(x + n)
→ c 6= 0.
Do đó
Q(x)
R(x)
= c hay Q(x) = c.R(x).
Vậy Q(x) = c.P(x).P(x + 1) · · · P(x + 2016).
Bài toán 1.13 (Poland MO). Cho đa thức P(x) có bậc n  1 có n nghiệm
thực x1, x2, . . . , xn phân biệt. Chứng minh rằng
1
P0(x1)
+
1
P0(x2)
+ . . . +
1
P0(xn)
= 0.
Lời giải. Đặt P(x) = a(x − x1)(x − x2) . . . (x − xn), a 6= 0,
suy ra
P0
(x) = P1(x) + P2(x) + . . . + Pn(x) với Pi(x) =
n
Y
j=1
(x − xj).
15
Ta thấy Pi(xj) = 0, ∀i 6= j. Suy ra P0
(xj) = Pj(xj) 6= 0, ∀j = 1, n.
Xét đa thức F(xi) =
n
P
i=1
Pi(xi)
P0(xi)
− 1 = 0 có bậc không quá n − 1.
Với i = 1, n, ta có F(xi) =
Pi(xi)
P0(xi)
− 1 = 0, suy ra F(x) có n nghiệm
phân biệt. Vậy nên F(x) ≡ 0.
Lại có hệ số của F(x) ứng với xn−1
bằng 0 nên
a
P0(x1)
+
a
P0(x2)
+ . . . +
a
P0(xn)
= 0.
Suy ra
1
P0(x1)
+
1
P0(x2)
+ . . . +
1
P0(xn)
= 0.
Tiếp theo xét bài toán sử dụng khá nhiều kĩ thuật: sử dụng định lí
Vi-et nhưng khéo léo trong việc đổi dấu các nghiệm nên không còn xuất
hiện (−1)n
, sử dụng đánh giá bất đẳng thức
∀x, y ≥ 1 ta luôn có xy +
1
x
+
1
y
≥
1
xy
+ x + y.
và sử dụng phương pháp quy nạp toán học.
Bài toán 1.14 (Moscow MO, 2011). Cho n ∈ N, n ≥ 3. Tìm tất cả các
đa thức hệ số thực f(x) = a0 + a1x + · · · + anxn
, (an 6= 0) có n nghiệm
không lớn hơn −1 và thỏa mãn điều kiện
a2
0 + a1an = a2
n + a0an−1.
Lời giải. Vì f(x) có n nghiệm không lớn hơn −1 nên
f(x) = a0 + a1x + . . . + anxn
= an(x + x1)(x + x2) . . . (x + xn).
Với mọi xi ≥ 1, ∀i = 1, n. Đặt















S1 = x1 + x2 + x3 + . . . + xn =
an−1
an
S2 = x1x2 + x2x3 + . . . + xn−1xn =
an−2
an
. . . . . . . . . . . . . . . . . . . . . . . . . . . . . . . . . . . .
Sn = x1x2x3 . . . xn =
a0
an
.
16
Ta có
a2
0 + a1an = a2
n + a0an−1
tương đương với

a0
an
2
+
a1
an
= 1 +
a0
an
.
an−1
an
.
S2
n + Sn−1 = 1 + SnS1 ⇔ Sn +
Sn−1
Sn
=
1
Sn
+ S1.
Do đó
x1x2 . . . xn +
1
x1
+ . . . +
1
xn
=
1
x1x2 . . . xn
+ x1 + . . . + xn.
mà
x1x2 . . . xn+
1
x1
+. . .+
1
xn
≥
1
x1x2 . . . xn
+x1+. . .+xn.( chứng minh theo qui nạp)
Dấu bằng xảy ra khi và chỉ khi có n − 1 số bằng 1.
Vậy đa thức cần tìm là f(x) = an(x + 1)n−1
(x + a) với a là hằng số
lớn hơn hoặc bằng 1.
Bài toán 1.15 (Olympic SV, 2000). Cho a, b ∈ R. Tìm tất cả các đa thức
P(x) thoả mãn điều kiện
xP(x − a) = (x − b)P(x), ∀x ∈ R.
Lời giải.
i) Khi a = 0, b = 0 thì P(x) tuỳ ý.
ii) Khi a = 0, b 6= 0 thì P(x) = 0 ∀x.
iii) Khi a 6= 0, b = 0 thì P(x) =const tuỳ ý.
iv) Khi a 6= 0, b 6= 0 thì:
a) Nếu
b
a
6∈ N, thì khi thay x = b vào ta được x = b − a là nghiệm.
Tương tự khi thay x = b − a thì sẽ có x = b − 2a là nghiệm,... Suy ra
P(x) = x, ∀x ∈ R.
b) Nếu
b
a
∈ N thì P(x) có x = a, x = 2a, . . . , x = (n−1)a là nghiệm.
Suy ra P(x) = (x − a)(x − 2a) . . . (x − (n − 1)a)Q(x).
17
Thế vào điều kiện bài ra, ta được
Q(x − a) = Q(x), ∀x ∈ R,
hay Q(x) = const .
Vậy nên
P(x) = (x − a)(x − 2a) . . . [x − (n − 1)a].
Bài toán 1.16 (Olympic 30-4, THPT chuyên Tiền Giang đề nghị). Gọi
xi, i = 1, 2011 là các nghiệm của đa thức P(x) = x2011
+ 2011x2000
+
a2009x2009
+ · · · + a0. Biết rằng x64
1 + x64
2 + · · · + x64
2011 = 2011.
Hãy xác định đa thức P(x).
Lời giải. Áp dụng định lí Viet, ta có
n−1
P
i=1
xi = −2011.
Áp dụng bất đẳng thức Cauchy, ta có
20112
=
 2011
X
i=1
xi
2
≤ 2011
 2011
X
i=1
x2
i

,
suy ra 20114
≤ 20112
2011
P
i=1
x2
i
2
.
Lại áp dụng bất đẳng thức Cauchy, ta có
2011
X
i=1
x2
i
2
≤ 2011
2011
X
i=1
x4
i

,
suy ra 20114
=
 2011
P
i=1
xi
4
≤ 20113
 2011
P
i=1
x4
i

.
Biến đổi tương tự, ta thu được
 2011
X
i=1
xi
8
≤ 20117
 2011
X
i=1
x8
i

.
Vậy nên
 2011
X
i=1
xi
64
≤ 201163
 2011
X
i=1
x64
i

= 201164
. (1.8)
Mặt khác,
n−1
P
i=1
xi = −2011, nên từ (1.8) suy ra bất đẳng thức Cauchy xảy
ra dấu bằng. Chứng tỏ các nghiệm bằng nhau và bằng -1. Do đó đa thức
cần tìm là P(x) = (x + 1)2011
.
18
1.4 Xác dịnh đa thức theo phép biến đổi vi phân
hàm
Trong phần này ta khảo sát một số dạng toán về xác định đa thức
theo phép biến đổi vi phân hàm.
Bài toán 1.17 (Olympic SV, 1993). Cho p(x) (6= const) là đa thức với
hệ số thực. Chứng minh rằng nếu hệ phương trình







x
R
0
p(t) sin tdt = 0,
x
R
0
p(t) cos tdt = 0.
có nghiệm thực thì số nghiệm thực chỉ có thể là hữu hạn.
Lời giải. Gọi p(k)
(t) là đạo hàm cấp k của p(t) (p0
(t) = p(t)) và ký
hiệu
Uk =
x
Z
0
p(k)
(t) sin tdt,
Vk =
x
Z
0
p(k)
(t) cos tdt.
Giả sử deg p = n. Suy ra Uk = 0, Vt = 0 nếu K  n. Sử dụng công thức
tích phân từng phần, ta thu được







Uk = −p(k)
(t) cos t
x
0
+
x
R
0
p(k+1)
(t) sin tdt
Vk = p(k)
(t) sin t
x
0
−
x
R
0
p(k+1)
(t) sin tdt.
Suy ra 
Uk = −p(k)
(t) cos t
x
0
+ Vk+1
Vk = p(k)
(t) sin t
x
0
− Uk+1
Ta có tiếp

Uk = −p(k)
(t) cos t
x
0
+ p(k+1)
(t) sin t
x
0
− Uk+2
Vk = p(k)
(t) sin t
x
0
+ p(k+1)
(t) cos t
x
0
− Vk+2
, ∀k ∈ N.
19









U0 = −
2k6n
P
k=0
p(2k)
(t) cos t
x
0
+
(2k+1)6n
P
k=0
p(2k+1)
(t) sin t
x
0
V0 =
2k6n
P
k=0
p(2k)
(t) sin t
x
0
+
(2k+1)6n
P
k=0
p(2k+1)
(t) cos t
x
0
.
đặt
p1(t) =
2k+16n
X
k=0
p(2k)
(t)
Suy ra deg p1 = n.
p2(t) =
2k+16n
X
k=0
p(2k+1)
(t).
Suy ra deg p2 = n − 1. Khi đó, (1) được viết dưới dạng

U0 = −p1(t) cos t
x
0
+ p2(t) sin t
x
0
V0 = p1(t) sin t
x
0
+ p2(t) cos
x
0
Gọi X là tập nghiệm của hệ đã cho, tức hệ

U0 = 0
V0 = 0.
Với mọi x ∈ X ta có

−p1(t) cos t|x
0 + p2 sin t|x
0 = 0
p1(t) sin t|x
0 + p2 cos t|x
0 = 0.
Đặt P1(0) = a, P2(0) = b. Khi đó

p2(x) sin x − p1(x) cos x = −a
p2(x) cos x + p1(x) sin x = b.
Suy ra
(p2(x) sin x − p1(x) cos x)2
+ (p2(x) cos x + p1(x) sin x)2
= a2
+ b2
.
Do đó
p2
1(x) + p2
2(x) − (a2
+ b2
) = 0.
Gọi Y là tập nghiệm của đa thức
Q(x) = p2
1(x) + p2
2(x) − (a2
+ b2
).
20
Suy ra X ⊂ Y . Từ deg Q = 2n suy ra |X| 6 |Y | 6 2n. Tức X chỉ có hữu
hạn phần tử.
Cách khác.
Ta có thể sử dụng số phức để giải bài toán. Viết lại hệ dưới dạng
F(x) :=
x
Z
0
P(t)eit
dt = 0.
Ta có
F0
(x) = P(x)eix
nên phương trình F0
(x) = 0 chỉ có hữu hạn nghiệm. Suy ra phương trình
F(x) = 0 có không quá hữu hạn nghiệm.
Bài toán 1.18 (Olympic SV,1994). a) Cho hàm số f : [a, b] → [a, b], với
a  b và thỏa mãn điều kiện
|f(x) − f(y)|  |x − y|, ∀x, y ∈ [a, b] và x 6= y.
Chứng minh rằng phương trình f(x) = x có duy nhất một nghiệm thuộc
[a, b].
b) Cho hàm số f(x) khả vi trên [a, b], có không điểm trên [a, b] và
thỏa mãn |f0
(x)|  |f(x)|, ∀x ∈ [a, b].
Chứng minh rằng f(x) = 0, ∀x ∈ [a, b].
Lời giải.
a) Xét hàm số g(x) = f(x) − x. Ta thấy g(x) liên tục trên [a, b]. Do
đó tồn tại x0 ∈ [a, b] sao cho
g(x0) = min
x∈[a,b]
g(x). (1.9)
Ta sẽ chứng minh rằng g(x0) = 0. Thật vậy, giả sử g(x0) 6= 0 và vì vậy,
f(x0) 6= x0.
Từ bất đẳng thức đã cho, ta có
f(f(x0)) − f(x0)  |f(x0) − x0|.
Suy ra
f(x0)  g(x0)
21
Điều này mâu thuẫn với (1.9), nghĩa là f(x0) = x0. Giả sử phương trình
f(x) = x còn có nghiệm x1 với x0 6= x1 ∈ [a, b]. Ta có

x1 6= x0
x1 ∈ [a, b].
Suy ra
|f(x1) − f(x0)| = |x1 − x0|,
Mâu thuẫn với bất đẳng thức đã cho.
Tóm lại, phương trình f(x) = x có duy nhất nghiệm trên [a, b].
b) Giả sử x0 là nghiệm của phương trình f(x) = 0 với x0 ∈ [a, b].
Theo khai triển Taylor tại x0, thì
f(x) = f(x0) + f0
(c)(x − x0) = f0
(c)(x − x0).
Xét khoảng đóng G :=
h
x0 −
1
2
, x0 +
1
2
i
∩ [a, b]. Vì f(x) khả vi trên [a, b]
nên f(x) đạt cực đại trên đoạn đóng G. Giả sử
|f(xm)| = max
x∈G
|f(x)|, xm ∈ G.
Suy ra
|f(xm)| = |f0
(cm)| |xm − x0| 6 |f(cm)| |xm − x0|
6
1
2
|f(cm)| 6
1
2
|f(xm)|.
Hay f(x) = 0 với mọi x ∈ G.
Như vậy, nếu tại một điểm trên [a, b] mà f(x) = 0 thì f(x) = 0 trên
toàn bộ lân cận với bán kính bằng 1/2 của điểm đó. Bằng việc xét các
điểm x0 khác nhau (mà tại đó f(x0) = 0) lan dần về hai phía của đoạn
[a, b] thì sau một số hữu hạn bước ta sẽ được f(x) = 0 với ∀x ∈ [a, b].
Bài toán 1.19 (Olympic SV, 1995). Xét đa thức
Pn(x) =
1
2nn!
dn
dxn
[x2
− 1)n
]
Chứng minh rằng nếu f(x) là đa thức bậc m (m  n) thì
1
Z
−1
f(x)Pn(x)dx = 0.
22
Lời giải. Sử dụng công thức tích phân từng phần
1
Z
−1
d(n)
(x2
− 1)n
dxn
f(x)dx =
=
d(n−1)
(x2
− 1)n
dxn−1
f(x)|1
−1 −
1
Z
−1
d(n−1)
(x2
− 1)n
dxn−1
f0
(x)dx
= · · · =
1
Z
−1
(x2
− 1)n
f(n)
(x)dx =
1
Z
−1
0dx = 0.
Bài toán 1.20 (Olympic SV, 1996). Cho g(x) là một đa thức bậc 1996.
Biết rằng, ứng với mọi x ∈ R, ta đều có
g(x + h) = g(x) + hg0
(x + h)θ(x, h),
trong đó θ(x, h) bị chặn và g00
(x) 6= 0. Tính lim
h→0
θ(x, h).
Lời giải.
Với x xác định, ta khai triển Taylor với đa thức f(x) = g(x + h) tại
h = 0 :
g(x + h) = g(x) + g0
(x)h +
g00
(x)
2
h2
+
g
000
(x)
3!
h3
+ · · · +
g1996
(x)h1996
1996!
(do f0
(0) = g0
(x), . . . , f(1996)
(0) = g(1996)
(x)).
Theo đề bài thì g(x + h) = g(x) + hg0
(x + hθ(x, h)). Do vậy
hg0
(t + hθ(x, h)) = hg0
(x) +
h2
2
g00
(x) + · · · +
h1996
1996!
g(1996)
(x).
Khai triển Taylor bậc 2 với hàm g0
(x + hθ(x, h)) tại điểm h = 0.
g0
(x + hθ(x, h)) = g0
(x) + g00
(x)hθ(x, h) + 0(hθ(x, h)),
nên
hg0
(x + hθ(x, h)) = hg0
(x) + h2
g00
(x)θ(x, h) + h0(hθ(x, h)) =
= hg0
(x) +
h2
2
g00
(x) + · · · +
h1996
1996!
g(1996)
(x).
23
Suy ra
g00
(x)θ(x, h) +
0(hθ(x, h))
h
=
=
12
2
g00
(x)+
h
3!
g
000
(x)+· · ·+
h1996
1996!
g(1996)
(x) = lim
h→0
g00
(x)θ(x, h) =
1
2
g00
(x).
Do g00
(x) 6= 0 và lim
n→0
θ(x, h) =
1
2
, nên
lim
n→0
θ(x, h) =
1
2
.
Bài toán 1.21 (Olympic SV, 1997). Chứng minh rằng, với mọi t  0,
phương trình x3
+ tx − 8 = 0 luôn có nghiệm dương duy nhất, ký hiệu là
x(t). Tính tích phân
t
Z
0
[x(t)]2
dt.
Lời giải.
Xét f(x) = x3
+ tx − 8. Ta có
f0
(x)  0, ∀x  0.
Mặt khác, ta có f(0) = −8  0 và lim
x→∞
f(x) = +∞. Vậy nên phương
trình f(x) = 0 có nghiệm dương duy nhất.
Từ phương trình x3
+ tx − 8 = 0, ta có t = x2
−
8
x
.
Khi t = 0 thì x = 2. Ta thu được t = 7 và x3
+ tx − 8 = (x − 1)(x2
+
x + t) = 0.
Suy ra x = 1 và
7
Z
0
[x(t)]2
dt = −
1
Z
2
x2
d

x2
−
8
x

=
2
Z
1
x2

2x +
8
x2

dx =
=
2
Z
1
(2x3
+ 8)dx =
x4
2
2
1
+ 8x
2
1

= −
31
2
.
24
Bài toán 1.22 (Olympic SV, 1998). Xét các đa thức P(x) với hệ số thực
thỏa mãn các điều kiện
P(0) = P(1) = 0,
1
Z
0
|P0
(x)|dx = 1.
Chứng minh rằng
|P(x)| 6
1
2
, ∀x ∈ [0, 1].
Lời giải. Ta sử dụng phương pháp chứng minh phản chứng. Giả sử
∃x0 ∈ [0, 1] sao cho |P(x0)| 
1
2
.
Do P(x) liên tục tại x0, nên suy ra
1
Z
0
|P0
(x)|dx =
x0
Z
0
|P0
(x)|dx +
1
Z
x0
|P0
(x)|dx 
x0
Z
0
P0
(x)dx +
1
Z
x0
P0
(x)dx
=|P(x0) − P(0)| + |P(1) − P(x0)|  2|P(x0)|  1,
mâu thuẫn với giả thiết
1
Z
0
|P0
(x)|dx = 1.
Vậy điều giả sử là sai.
Vậy nên
|P(x)| 6
1
2
, ∀x ∈ [0, 1].
Bài toán 1.23 (Olympic SV, 1999). Giả sử đa thức với hệ số thực
P(x) = a0 + a1x + · · · + anxn
có n nghiệm thực phân biệt. Chứng minh rằng
ak−1ak+1  a2
k, ∀k ∈ {1, 2, . . . , n − 1}.
Lời giải.
25
Ta sẽ chứng minh rằng
[Q0
(x)]2
− Q(x)Q00
(x)  0, ∀x ∈ R, (1.10)
ứng với Q(x) ∈ R(x), deg Q(x) = m và Q(x) có m nghiệm thực đơn.
Ta có
Q(x) = a
m
Y
i=1
(x − αi), αi 6= αj (i 6= j).
Suy ra
Q0
(x)
Q(x)
=
m
X
i=1
1
x − αi
và
[Q0
(x)]2
− Q(x)Q00
(x)
Q2(x)
=
m
X
i=1
1
(x − αi)2
. (1.11)
a) Nếu với t ∈ R mà Q(t) = 0 thì
[Q0
(t)]2
− Q(t)Q00
(t) = [Q0
(t)]2
 0
(do Q0
(t) 6= 0 và do t là nghiệm đơn).
b) Nếu với t ∈ R mà Q(t) 6= 0 thì từ (1.11) suy ra (1.10).
Bây giờ ta xét đa thức
Q(x) = P(k)
(x), k = 0, 1, . . . , n − 1.
Các đa thức đó đều có nghiệm thực đơn (định lý Role). Suy ra
P(k−1)
(0)P(k+1)
(0)  [P(k)
]2
hay
(k − 1)!ak−1(k + 1)!ak+1  (akk!)2
nên
ak−1ak+1  (ak)2 k
k + 1
 a2
k.
Bài toán 1.24 (Olympic SV, 2001). Cho hàm số f(x) khả vi trên đoạn
[a, b] và thoả mãn điều kiện
[f(x)]2
+ [f0
(x)]2
 0, ∀x ∈ [a, b].
26
Chứng minh rằng số các nghiệm của phương trình f(x) = 0 trên đoạn
[a, b] là hữu hạn.
Lời giải. Giả sử ngược lại, phương trình f(x) = 0 có vô số nghiệm
{xn} ∈ [a, b], n = 1, 2, 3, . . . Khi đó, tồn tại dãy con {xnk
} → α ∈ [a, b].
Do f(x) liên tục nên f(α) = 0. Từ giả thiết [f(x)]2
+ [f0
(x)]2
 0, ∀x ∈
[a, b], suy ra f0
(α) 6= 0.
Mặt khác,
f0
(α) = lim
x→α
f(x) − f(α)
x − α
6= 0.
Điều này chứng tỏ f(x) 6= 0 trong một lân cận nào đó của điểm α, mâu
thuẫn với giả thiết α là điểm tụ của dãy {xn}n=1,2,3,....
27
Chương 2
Ước lượng đa thức
2.1 Đa thức Chebyshev và các tính chất
Panuty Chebyshev là tên của một nhà toán học người Nga. Các kết
qủa về dãy đa thức trực giao (orthogonal polynomials) có liên hệ sâu sắc
đến công thức de Moivre (de Moivre’s formula). Các dãy đa thức trực giao
này hoàn toàn có thể xác định được bằng công thức truy hồi giống như
dãy số Fibonacci và dãy số Lucas.
Đa thức này gồm có hai loại là:
Ta ký hiệu đa thức Chebyshev loại I là Tn. Chữ T được chọn làm ký
hiệu vì tên của Chebyshev trong tiếng Pháp là Tchebycheff và trong tiếng
Đức là Tschebyscheff.
Ta ký hiệu đa thức Chebyshev loại II là Un.
Định nghĩa 2.1 (Đa thức Chebyshev loại I). Các đa thức Tn (x) được xác
định bởi:

T0 (x) = 1; T1 (x) = x
Tn+1 (x) = 2x.Tn (x) − Tn−1 (x) , n ≥ 1 được gọi là đa thức
Chebyshev loại I.
Định nghĩa 2.2 (Đa thức Chebyshev loại II). Các đa thức Un (x) được
xác định bởi:

U0 (x) = 1; U1 (x) = 2x
Un+1 (x) = 2x.Un (x) − Un−1 (x) , n ≥ 1 được gọi là đa
thức Chebyshev loại II.
Bằng phương pháp quy nạp toán học, ta dễ dàng chứng minh được:
Tn (cos α) = cos nα, ∀α ∈ R; Un (cos α) =
sin (n + 1) α
sin α
, ∀α 6= kπ, k ∈ Z.
Tính chất 2.1. Tn (x) = cos (n arccos x) , ∀x ∈ [−1; 1] ;
28
Un (x) =
sin (n arccos x)
√
1 − x2
, ∀x ∈ (−1; 1) .
Tính chất 2.2. Tn (x) , Un (x) ∈ Z [x] có bậc là n và hệ số cao nhất tương
ứng là 2n−1
và 2n
.
Tính chất 2.3. Tn (x) , U (x) là các hàm số chẵn khi n chẵn và là các hàm
số lẻ khi n lẻ.
Tính chất 2.4. Tn (x) , Un (x) có đúng n nghiệm thực phân biệt tương
ứng là: cos
(2k + 1) π
2n
, k = 0; n − 1 và cos
kπ
n + 1
, k = 1, n.
Chứng minh. Do x ∈ [−1; 1] nên ta đặt x = cos α với α ∈ [0; π].
Tn (x) = 0 tương đương Tn (cos α) = 0 suy ra cos nα = 0 do đó nα =
π
2
+ kπ hay α =
π
2n
+
k
n
π.
Do α ∈ [0; π] nên 0 ≤
π
2n
+
k
n
π ≤ π ⇔ 0 ≤
1
2n
+
k
n
≤ 1
⇔ −
1
2
≤ k ≤
2n − 1
2
= n −
1
2
.
Mà k ∈ Z nên k = 0, n − 1.
Vậy Tn (x) có đúng n nghiệm thực phân biệt tương ứng là:
cos
(2k + 1) π
2n
, k = 0; n − 1. Ta chứng minh tương tự cho trường hợp
của Un (x).
Tính chất 2.5. |Tn (x)| ≤ 1, ∀x ∈ [−1; 1] và |Tn (x)| = 1 có đúng n + 1
điểm xk = cos
kπ
n
, k = 0; n.
Chứng minh. Theo cách đặt trên thì
|Tn (x)| = |Tn (cos α)| = |cos nα| ≤ 1, ∀x ∈ [−1; 1].
|Tn (x)| = 1 ⇔ |cos nα| = 1 ⇔ sin nα = 0 ⇔ nα = kπ ⇔ α =
kπ
n
.
Do α ∈ [0; π] nên 0 ≤
kπ
n
≤ π ⇔ 0 ≤ k ≤ n. Mà k ∈ Z nên k = 0; n.
Lưu ý. Các điểm cos
kπ
n
, k = 0; n nói ở trên là các điểm luân phiên
Chebyshev (gọi tắt là các luân điểm).
Nhận xét 2.1. Tn

cos
kπ
n

= (−1)k
.
29
Tính chất 2.6. Tn

1
2

x +
1
x

=
1
2

xn
+
1
xn

, ∀n ∈ N, ∀x 6= 0;
Un

1
2

x +
1
x

=
xn+1
−
1
xn+1
x −
1
x
, ∀n ∈ N, ∀x 6= 0; ±1.
Chứng minh. Với n = 0, n = 1 ta có
T0

1
2

x +
1
x

= 1; T1

1
2

x +
1
x

=
1
2

x +
1
x

luôn đúng.
Giả sử mệnh đề trên đúng đến n. Áp dụng công thức truy hồi của Tn
ta có:
Tn+1

1
2

x +
1
x

= 2.
1
2

x +
1
x

Tn

1
2

x +
1
x

−Tn−1

1
2

x +
1
x

=
1
2

x +
1
x
 
xn
+
1
xn

−

xn−1
+
1
xn−1

=
1
2

xn+1
+
1
xn+1

Ta chứng minh tương tự cho trường hợp của Un (x).
Hệ quả 2.1. ∀n ∈ N ta có:
Tn (x) =

x +
√
x2 − 1
n
+

x −
√
x2 − 1
n
2
;
Un (x) =

x +
√
x2 − 1
n
−

x −
√
x2 − 1
n
2
√
x2 − 1
.
Tính chất 2.7. (Tn (x) ; Un (x)) là cặp nghiệm của phương trình Pell đa
thức:
P2
(x) − x2
− 1

Q2
(x) = 1.
Tính chất 2.8. Tn 1 − 2x2

= (−1)n
T2n; Un 1 − 2x2

x = (−1)n
U2n+1 (x)
Tính chất 2.9. Un (x) = xUn−1 (x) + Tn−1 (x) , ∀x ∈ R, n ∈ N∗
.
Tính chất 2.10. Tn+1 (x) = xTn (x) − 1 − x2

Un (x) , ∀x ∈ R, n ∈ N.
Tính chất 2.11. Tn+m (x)+T|n−m| (x) = 2Tn (x) Tm (x) ; ∀x ∈ R, ∀n, m ∈
N.
Tính chất 2.12. Tm (Tn (x)) = Tmn (x) , ∀x ∈ R, ∀m, n ∈ N.
Tính chất 2.13. Un (x) =
1
n + 1
T0
n+1 (x).
30
Tính chất 2.14. |U (x)| ≤ n ∀x ∈ [−1; 1]; |T0
n (x)| ≤ n2
∀x ∈ [−1; 1].
Tính chất 2.15.
dTn
dx
= nUn−1;
dUn
dx
=
(n + 1) Tn+1 − xUn
x2 − 1
;
d2
Tn
dx2
= n.
nTn − xUn−1
x2 − 1
= n.
(n + 1) Tn − Un
x2 − 1
.
Tổng quát:
dp
Tn
dxp
x=±1
= (±1)n+p
p−1
Q
k=0
n2
− k2
2k + 1
.
Nhận xét 2.2.
d2
Tn
dx2
x=1
=
n4
− n2
3
;
d2
Tn
dx2
x=−1
= (−1)n n4
− n2
3
.
Chứng minh. T00
(1) = lim
x→1
n.
nTn − xUn−1
x2 − 1
= lim
x→1
n.
nTn − xUn−1
x − 1
x + 1
=
n.
lim
x→1
nTn − xUn−1
x − 1
lim
x→1
(x + 1)
=
n
2
lim
x→1
nTn − xUn−1
x − 1
Áp dụng quy tắc L’Hospital ta có:
T00
(1) =
n
2
lim
x→1
d
dx
(nTn − xUn−1)
d
dx
(x − 1)
=
n
2
lim
x→1
d
dx
(nTn − xUn−1)
=
n
2
lim
x→1

n2
Un−1 − Un−1 − x
d
dx
(Un−1)

=
n
2

n2
Un−1 (1) − Un−1 (1) − lim
x→1
d
d
dx
(Un−1)

=
n4
2
−
n2
2
−
1
2
lim
x→1
d
dx
(nUn−1)
=
n4
2
−
n2
2
−
T00
n (1)
2
=
n4
− n2
3
.
Trường hợp còn lại chứng minh tương tự.
Tính chất 2.16. 1 − x2

T00
n (x)−xT0
n (x)+n2
Tn (x) = 0, ∀x ∈ R, ∀n ∈
N.
Tính chất 2.17.
R
Undx =
Tn+1
n + 1
+ C;
31
Z
Tndx =
1
2

Tn+1
n + 1
−
Tn−1
n − 1

+ C =
nTn+1
n2 − 1
−
xTn
n − 1
+ C.
Do dung lượng quy định của luận văn nên tác giả không say sưa vào
việc chứng minh các tính chất trên mặc dù việc chứng minh này rất thú
vị. Phần lớn các tính chất trên đều được chứng minh theo quy nạp kết
hợp các công thức truy hồi.
2.2 Các dạng toán liên quan đến đa thức Cheby-
shev
Trong các dạng toán về ước lượng đa thức, đa thức Chebyshev đóng
vai trò rất quan trọng. Nó vừa là trường hợp xảy ra dấu đẳng thức của
nhiều bài toán ước lượng đa thức, vừa là đa thức bổ trợ trong các lời
giải và chứng minh.Ta giải quyết một số bài toán liên quan đến đa thức
Chebyshev.
Bài toán 2.1. Cho đa thức f (x) = 4x3
+ (m + 3) x2
+ mx. Tìm m ∈ R
để |f (x)| ≤ 1, ∀x ∈ [−1; 1]
Lời giải. Do f (x) là đa thức bậc 3 và kèm theo điều kiện x ∈ [−1; 1] nên
ta có thể liên hệ đến các điểm là các luân điểm của đa thức Chebyshev bậc
3 là xk = cos
kπ
3
(k = 0; 1; 2; 3). Cụ thể là x0 = cos 0 = 1; x1 = cos
π
3
=
1
2
;
x2 = cos
2π
3
= −
1
2
; x3 = cos π = −1.
Ta có:













|f (1)| ≤ 1
|f (−1)| ≤ 1
f

−
1
2

≤ 1
f

1
2

≤ 1
⇒













|2m + 7| ≤ 1
|−1| ≤ 1
−
1
2
+
m + 3
4
−
m
2
≤ 1
1
2
+
m + 3
4
+
m
2
≤ 1
⇒











|2m + 7| ≤ 1
1 − m
4
≤ 1
3m + 5
4
≤ 1
Suy ra
(
−1 ≤ 2m + 7 ≤ 1
−4 ≤ 1 − m ≤ 4
−4 ≤ 3m + 5 ≤ 4
32
Tải bản FULL (77 trang): https://bit.ly/3fQM1u2
Dự phòng: fb.com/KhoTaiLieuAZ
Do đó





−4 ≤ m ≤ −3
−3 ≤ m ≤ 5
−3 ≤ m ≤
1
3
nên m = −3
Đảo lại, khi m = −3 thì f (x) = 4x3
− 3x.
Đặt x = cos α với α ∈ [0; π] .
Ta có f (x) = cos 3α. Suy ra |f (x)| = |cos 3α| ≤ 1.
Bài toán 2.2. Cho đa thức f (x) = 4x3
+ ax2
+ bx + c. Tìm a, b, c ∈ R
để |f (x)| ≤ 1, ∀x ∈ [−1; 1].
Lời giải. Đây là một bài khá hấp dẫn, tôi xin trình bày với nhiều lời giải
khác nhau.
Cách 1. Bài toán này tương tự với cách ra đề và cách suy luận như
bài toán trên. Tuy nhiên, bài toán này có đến 3 tham số đòi hỏi chúng ta
phải có những đánh giá sâu sắc hơn. Dựa vào cách suy luận ở bài toán
trên, ta để ý đến các luân điểm của đa thức Chebyshev bậc 3 là:
x0 = cos 0 = 1; x1 = cos
π
3
=
1
2
; x2 = cos
2π
3
= −
1
2
; x3 = cos π =
−1.
Ta có đánh giá sau:













|f (1)| ≤ 1
|f (−1)| ≤ 1
f

1
2

≤ 1
f

−
1
2

≤ 1
⇒













|4 + a + b + c| ≤ 1
|−4 + a − b + c| ≤ 1
1
2
+
a
4
+
b
2
+ c ≤ 1
−
1
2
+
a
4
−
b
2
+ c ≤ 1
Suy ra





−5 ≤ a + b + c ≤ −3 (1)
3 ≤ a − b + c ≤ 5 (2)
−6 ≤ a + 2b + 4c ≤ 2 (3)
−2 ≤ a − 2b + 4c ≤ 6 (4)
Sử dụng (1) và (2) ta có:

a + b + c ≤ −3
a − b + c ≥ 3 ⇒ b ≤ −3.
Sử dụng (3) và (4) ta có:

a + 2b + 4c ≥ −6
a − 2b + 4c ≤ 6 ⇒ b ≥ −3.
Từ hai đánh giá trên, ta suy ra b = −3.
Thay b = −3 vào hệ bất phương trình ở trên, ta được:
33
Tải bản FULL (77 trang): https://bit.ly/3fQM1u2
Dự phòng: fb.com/KhoTaiLieuAZ





−2 ≤ a + c ≤ 0
0 ≤ a + c ≤ 2
−8 ≤ a + 4c ≤ 0
0 ≤ a + 4c ≤ 8
⇒

a + c = 0
a + 4c = 0 ⇔ a = c = 0.
Đảo lại, khi a = 0; b = −3; c = 0 thì f (x) = 4x3
− 3x.
Đặt x = cos α với α ∈ [0; π] ta cóf (x) = cos 3α. Suy ra |f (x)| =
|cos 3α| ≤ 1.
Cách 2. Để cho việc đánh giá và tìm các giá trị của a; b; c được nhanh
hơn ta cần quan tâm đến bất đẳng thức quen thuộc |a1 + a2 + ... + an| ≤
|a1| + |a2| + ... + |an| và dấu đẳng thức có được khi các ai cùng dấu. Ta
dự đoán đích đến là a = 0; b = −3; c = 0.
Quay lại với bài toán.
Ta có:
|8 + 2b| ≤ |4 + a + b + c| + |4 + b − a − c| ≤ 2. Suy ra |4 + b| ≤ 1.
Lại có: |−1 − b| ≤ −
1
2
−
1
4
a −
1
2
b − c + −
1
2
+
1
4
a −
1
2
b + c ≤ 2.
Vấn đề ở đây là ta tìm được giá trị của b.
Bây giờ nhận thấy 3 ≤ |4 + b| + |−1 − b| ≤ 1 + 2 = 3.
Vậy thì dấu bằng xảy ra ở tất cả các bất đẳng thức trên là
b = −3; a = c = 0.
Đảo lại, khi a = 0; b = −3; c = 0 thì f (x) = 4x3
− 3x.
Đặt x = cos α với α ∈ [0; π] ta cóf (x) = cos 3α.
Suy ra |f (x)| = |cos 3α| ≤ 1.
Cách 3.
Do xem xét trên đoạn [−1; 1] nên ta cần quan tâm đến giá trị lớn
nhất của hàm số |f (x)| để việc đánh giá dưới đây được thuận lợi.
Quay lại bài toán
Giả sử tồn tại các số a, b, c thỏa mãn giả thiết bài toán và đặt
M = max
x∈[−1;1]
|f (x)|. Ta có
|f (1)| = |4 + a + b + c| ; |f (−1)| = |−4 + a − b + c| ;
f

1
2

=
1
2
+
a
4
+
b
2
+ c ; f

−
1
2

= −
1
2
+
a
4
−
b
2
+ c .
Với cách đặt ở trên, ta suy ra
34
6732069

More Related Content

What's hot

Bộ sưu tập bất đẳng thức của võ quốc bá cẩn
Bộ sưu tập bất đẳng thức của võ quốc bá cẩnBộ sưu tập bất đẳng thức của võ quốc bá cẩn
Bộ sưu tập bất đẳng thức của võ quốc bá cẩn
Thế Giới Tinh Hoa
 
Chuyên đề phương tích và ứng dụng
Chuyên đề phương tích và ứng dụngChuyên đề phương tích và ứng dụng
Chuyên đề phương tích và ứng dụnglovemathforever
 
Toan a1 -_bai_giang
Toan a1 -_bai_giangToan a1 -_bai_giang
Toan a1 -_bai_giangxuanhoa88
 
Cơ sở giải tích - Độ đo
Cơ sở giải tích - Độ đo Cơ sở giải tích - Độ đo
Cơ sở giải tích - Độ đo
Bui Loi
 
Topo daicuong1[1]
Topo daicuong1[1]Topo daicuong1[1]
Topo daicuong1[1]
Bui Loi
 
Luận văn: Một số phương pháp giải phương trình hàm, HOT, 9đ
Luận văn: Một số phương pháp giải phương trình hàm, HOT, 9đLuận văn: Một số phương pháp giải phương trình hàm, HOT, 9đ
Luận văn: Một số phương pháp giải phương trình hàm, HOT, 9đ
Dịch vụ viết bài trọn gói ZALO: 0909232620
 
Diophantine equations Phương trình diophant
Diophantine equations Phương trình diophantDiophantine equations Phương trình diophant
Diophantine equations Phương trình diophant
Bui Loi
 
Luận văn: Bất đẳng thức trong lớp hàm siêu việt, HAY, 9đ
Luận văn: Bất đẳng thức trong lớp hàm siêu việt, HAY, 9đLuận văn: Bất đẳng thức trong lớp hàm siêu việt, HAY, 9đ
Luận văn: Bất đẳng thức trong lớp hàm siêu việt, HAY, 9đ
Dịch Vụ Viết Bài Trọn Gói ZALO 0917193864
 
Các phương pháp hay giải Phuong trinh-vo-ty
Các phương pháp hay giải Phuong trinh-vo-tyCác phương pháp hay giải Phuong trinh-vo-ty
Các phương pháp hay giải Phuong trinh-vo-tyroggerbob
 
45099385 bai-tap-do-do-tich-phan (1)
45099385 bai-tap-do-do-tich-phan (1)45099385 bai-tap-do-do-tich-phan (1)
45099385 bai-tap-do-do-tich-phan (1)
Vinh Phan
 
Một số dạng toán về đa thức qua các kỳ thi Olympic
Một số dạng toán về đa thức qua các kỳ thi OlympicMột số dạng toán về đa thức qua các kỳ thi Olympic
Một số dạng toán về đa thức qua các kỳ thi Olympic
Man_Ebook
 
Gt khong gian_metric Nguyen Hoang
Gt khong gian_metric Nguyen HoangGt khong gian_metric Nguyen Hoang
Gt khong gian_metric Nguyen Hoang
Bui Loi
 
Ứng dụng phương tích và trục đẳng phương vào bài toán hình học phẳng
Ứng dụng phương tích và trục đẳng phương vào bài toán hình học phẳng Ứng dụng phương tích và trục đẳng phương vào bài toán hình học phẳng
Ứng dụng phương tích và trục đẳng phương vào bài toán hình học phẳng
Bui Loi
 
Bt dai so hoang
Bt dai so hoangBt dai so hoang
Bt dai so hoang
Hoanghl Lê
 
Chuyên đề phương tích và ứng dụng
Chuyên đề phương tích và ứng dụngChuyên đề phương tích và ứng dụng
Chuyên đề phương tích và ứng dụng
Ngo Quang Viet
 
Chuong 2 dai so tuyen tinh 2
Chuong 2   dai so tuyen tinh 2Chuong 2   dai so tuyen tinh 2
Chuong 2 dai so tuyen tinh 2
Trương Huỳnh
 
Một số tính chất của vành giao hoán artin
Một số tính chất của vành giao hoán artinMột số tính chất của vành giao hoán artin
Một số tính chất của vành giao hoán artin
NOT
 
Luận văn: Các bài toán về hệ thức lượng trong tam giác, HOT, 9đ
Luận văn: Các bài toán về hệ thức lượng trong tam giác, HOT, 9đLuận văn: Các bài toán về hệ thức lượng trong tam giác, HOT, 9đ
Luận văn: Các bài toán về hệ thức lượng trong tam giác, HOT, 9đ
Dịch vụ viết bài trọn gói ZALO 0917193864
 
Phương Tích - Trục Đẳng Phương
Phương Tích - Trục Đẳng PhươngPhương Tích - Trục Đẳng Phương
Phương Tích - Trục Đẳng Phương
Nhập Vân Long
 

What's hot (20)

Bộ sưu tập bất đẳng thức của võ quốc bá cẩn
Bộ sưu tập bất đẳng thức của võ quốc bá cẩnBộ sưu tập bất đẳng thức của võ quốc bá cẩn
Bộ sưu tập bất đẳng thức của võ quốc bá cẩn
 
Chuyên đề phương tích và ứng dụng
Chuyên đề phương tích và ứng dụngChuyên đề phương tích và ứng dụng
Chuyên đề phương tích và ứng dụng
 
Toan a1 -_bai_giang
Toan a1 -_bai_giangToan a1 -_bai_giang
Toan a1 -_bai_giang
 
Cơ sở giải tích - Độ đo
Cơ sở giải tích - Độ đo Cơ sở giải tích - Độ đo
Cơ sở giải tích - Độ đo
 
Hinh hoc-affine
Hinh hoc-affineHinh hoc-affine
Hinh hoc-affine
 
Topo daicuong1[1]
Topo daicuong1[1]Topo daicuong1[1]
Topo daicuong1[1]
 
Luận văn: Một số phương pháp giải phương trình hàm, HOT, 9đ
Luận văn: Một số phương pháp giải phương trình hàm, HOT, 9đLuận văn: Một số phương pháp giải phương trình hàm, HOT, 9đ
Luận văn: Một số phương pháp giải phương trình hàm, HOT, 9đ
 
Diophantine equations Phương trình diophant
Diophantine equations Phương trình diophantDiophantine equations Phương trình diophant
Diophantine equations Phương trình diophant
 
Luận văn: Bất đẳng thức trong lớp hàm siêu việt, HAY, 9đ
Luận văn: Bất đẳng thức trong lớp hàm siêu việt, HAY, 9đLuận văn: Bất đẳng thức trong lớp hàm siêu việt, HAY, 9đ
Luận văn: Bất đẳng thức trong lớp hàm siêu việt, HAY, 9đ
 
Các phương pháp hay giải Phuong trinh-vo-ty
Các phương pháp hay giải Phuong trinh-vo-tyCác phương pháp hay giải Phuong trinh-vo-ty
Các phương pháp hay giải Phuong trinh-vo-ty
 
45099385 bai-tap-do-do-tich-phan (1)
45099385 bai-tap-do-do-tich-phan (1)45099385 bai-tap-do-do-tich-phan (1)
45099385 bai-tap-do-do-tich-phan (1)
 
Một số dạng toán về đa thức qua các kỳ thi Olympic
Một số dạng toán về đa thức qua các kỳ thi OlympicMột số dạng toán về đa thức qua các kỳ thi Olympic
Một số dạng toán về đa thức qua các kỳ thi Olympic
 
Gt khong gian_metric Nguyen Hoang
Gt khong gian_metric Nguyen HoangGt khong gian_metric Nguyen Hoang
Gt khong gian_metric Nguyen Hoang
 
Ứng dụng phương tích và trục đẳng phương vào bài toán hình học phẳng
Ứng dụng phương tích và trục đẳng phương vào bài toán hình học phẳng Ứng dụng phương tích và trục đẳng phương vào bài toán hình học phẳng
Ứng dụng phương tích và trục đẳng phương vào bài toán hình học phẳng
 
Bt dai so hoang
Bt dai so hoangBt dai so hoang
Bt dai so hoang
 
Chuyên đề phương tích và ứng dụng
Chuyên đề phương tích và ứng dụngChuyên đề phương tích và ứng dụng
Chuyên đề phương tích và ứng dụng
 
Chuong 2 dai so tuyen tinh 2
Chuong 2   dai so tuyen tinh 2Chuong 2   dai so tuyen tinh 2
Chuong 2 dai so tuyen tinh 2
 
Một số tính chất của vành giao hoán artin
Một số tính chất của vành giao hoán artinMột số tính chất của vành giao hoán artin
Một số tính chất của vành giao hoán artin
 
Luận văn: Các bài toán về hệ thức lượng trong tam giác, HOT, 9đ
Luận văn: Các bài toán về hệ thức lượng trong tam giác, HOT, 9đLuận văn: Các bài toán về hệ thức lượng trong tam giác, HOT, 9đ
Luận văn: Các bài toán về hệ thức lượng trong tam giác, HOT, 9đ
 
Phương Tích - Trục Đẳng Phương
Phương Tích - Trục Đẳng PhươngPhương Tích - Trục Đẳng Phương
Phương Tích - Trục Đẳng Phương
 

Similar to Một số dạng toán về đa thức qua các kỳ thi Olympic 6732069.pdf

Luận văn: Đa tạp tâm của hệ tam phân mũ không đều, HOT
Luận văn: Đa tạp tâm của hệ tam phân mũ không đều, HOTLuận văn: Đa tạp tâm của hệ tam phân mũ không đều, HOT
Luận văn: Đa tạp tâm của hệ tam phân mũ không đều, HOT
Dịch Vụ Viết Bài Trọn Gói ZALO 0917193864
 
Luận văn: Đa tạp tâm của hệ tam phân mũ không đều, HAY, 9đ
Luận văn: Đa tạp tâm của hệ tam phân mũ không đều, HAY, 9đLuận văn: Đa tạp tâm của hệ tam phân mũ không đều, HAY, 9đ
Luận văn: Đa tạp tâm của hệ tam phân mũ không đều, HAY, 9đ
Dịch vụ viết bài trọn gói ZALO 0917193864
 
Đề tài: Tính ổn định của lớp phương trình hàm với cặp biến tự do
Đề tài: Tính ổn định của lớp phương trình hàm với cặp biến tự doĐề tài: Tính ổn định của lớp phương trình hàm với cặp biến tự do
Đề tài: Tính ổn định của lớp phương trình hàm với cặp biến tự do
Dịch vụ viết bài trọn gói ZALO 0917193864
 
Bat đang thức trong so hoc và m t so Dạng toán liên quan.docx
Bat đang thức trong so hoc và m t so Dạng toán liên quan.docxBat đang thức trong so hoc và m t so Dạng toán liên quan.docx
Bat đang thức trong so hoc và m t so Dạng toán liên quan.docx
DV Viết Luận văn luanvanmaster.com ZALO 0973287149
 
Luận văn: Lớp bài toán tìm giá trị lớn nhất giá trị nhỏ nhất, HOT
Luận văn: Lớp bài toán tìm giá trị lớn nhất giá trị nhỏ nhất, HOTLuận văn: Lớp bài toán tìm giá trị lớn nhất giá trị nhỏ nhất, HOT
Luận văn: Lớp bài toán tìm giá trị lớn nhất giá trị nhỏ nhất, HOT
Dịch vụ viết bài trọn gói ZALO: 0909232620
 
Đề tài: Phương pháp giải hệ phương trình và hệ bất phương trình
Đề tài: Phương pháp giải hệ phương trình và hệ bất phương trìnhĐề tài: Phương pháp giải hệ phương trình và hệ bất phương trình
Đề tài: Phương pháp giải hệ phương trình và hệ bất phương trình
Dịch vụ viết bài trọn gói ZALO 0917193864
 
Luận văn: Phương trình phi tuyến không chỉnh loại J - đơn điệu, 9đ
Luận văn: Phương trình phi tuyến không chỉnh loại J - đơn điệu, 9đLuận văn: Phương trình phi tuyến không chỉnh loại J - đơn điệu, 9đ
Luận văn: Phương trình phi tuyến không chỉnh loại J - đơn điệu, 9đ
Dịch vụ viết bài trọn gói ZALO 0917193864
 
SH_Lien_ND_Dinh ly thang du Trung Hoa_VP_2016_08_16.pdf
SH_Lien_ND_Dinh ly thang du Trung Hoa_VP_2016_08_16.pdfSH_Lien_ND_Dinh ly thang du Trung Hoa_VP_2016_08_16.pdf
SH_Lien_ND_Dinh ly thang du Trung Hoa_VP_2016_08_16.pdf
NguyenTanBinh4
 
Định lý zsigmondy và Tính chất số học của đa thức.docx
Định lý zsigmondy và Tính chất số học của đa thức.docxĐịnh lý zsigmondy và Tính chất số học của đa thức.docx
Định lý zsigmondy và Tính chất số học của đa thức.docx
DV Viết Luận văn luanvanmaster.com ZALO 0973287149
 
Bat Đang Thức Và Bài Toán Cực Tr± Trong L P Các Đa Thức Và Phân Thức H So Ngu...
Bat Đang Thức Và Bài Toán Cực Tr± Trong L P Các Đa Thức Và Phân Thức H So Ngu...Bat Đang Thức Và Bài Toán Cực Tr± Trong L P Các Đa Thức Và Phân Thức H So Ngu...
Bat Đang Thức Và Bài Toán Cực Tr± Trong L P Các Đa Thức Và Phân Thức H So Ngu...
DV Viết Luận văn luanvanmaster.com ZALO 0973287149
 
Một số phương pháp tìm cực trị của các hàm phân thức Sinh bởi số tự nhiên.docx
Một số phương pháp tìm cực trị của các hàm phân thức Sinh bởi số tự nhiên.docxMột số phương pháp tìm cực trị của các hàm phân thức Sinh bởi số tự nhiên.docx
Một số phương pháp tìm cực trị của các hàm phân thức Sinh bởi số tự nhiên.docx
DV Viết Luận văn luanvanmaster.com ZALO 0973287149
 
Phương Pháp Lư Ng Giác Giải Phương Trình Đa Thức Và M T So Dạng Toán.docx
Phương Pháp Lư Ng Giác Giải Phương Trình Đa Thức Và M T So Dạng Toán.docxPhương Pháp Lư Ng Giác Giải Phương Trình Đa Thức Và M T So Dạng Toán.docx
Phương Pháp Lư Ng Giác Giải Phương Trình Đa Thức Và M T So Dạng Toán.docx
DV Viết Luận văn luanvanmaster.com ZALO 0973287149
 
Luận văn: Bài toán cực trị với đa thức đối xứng ba biến, HAY - Gửi miễn phí q...
Luận văn: Bài toán cực trị với đa thức đối xứng ba biến, HAY - Gửi miễn phí q...Luận văn: Bài toán cực trị với đa thức đối xứng ba biến, HAY - Gửi miễn phí q...
Luận văn: Bài toán cực trị với đa thức đối xứng ba biến, HAY - Gửi miễn phí q...
Dịch vụ viết bài trọn gói ZALO: 0909232620
 
Đề tài: Lớp bất đẳng thức, bài toán cực trị với đa thức đối xứng, 9đ
Đề tài: Lớp bất đẳng thức, bài toán cực trị với đa thức đối xứng, 9đĐề tài: Lớp bất đẳng thức, bài toán cực trị với đa thức đối xứng, 9đ
Đề tài: Lớp bất đẳng thức, bài toán cực trị với đa thức đối xứng, 9đ
Dịch vụ viết bài trọn gói ZALO: 0909232620
 
Luận văn: Giải hình thức các phương trình tích phân Volterra, HAY
Luận văn: Giải hình thức các phương trình tích phân Volterra, HAYLuận văn: Giải hình thức các phương trình tích phân Volterra, HAY
Luận văn: Giải hình thức các phương trình tích phân Volterra, HAY
Dịch vụ viết bài trọn gói ZALO 0917193864
 
Bài giảng chi tiết giải tích 1 07 2013 bo môn toán
Bài giảng chi tiết giải tích 1  07 2013 bo môn toánBài giảng chi tiết giải tích 1  07 2013 bo môn toán
Bài giảng chi tiết giải tích 1 07 2013 bo môn toán
Laurent Koscielny
 
Luận văn: Phương pháp giải hệ phương trình trong toán THPT
Luận văn: Phương pháp giải hệ phương trình trong toán THPTLuận văn: Phương pháp giải hệ phương trình trong toán THPT
Luận văn: Phương pháp giải hệ phương trình trong toán THPT
Dịch Vụ Viết Bài Trọn Gói ZALO 0917193864
 
Luận văn: Giải hệ phương trình trong chương trình toán THPT, HOT
Luận văn: Giải hệ phương trình trong chương trình toán THPT, HOTLuận văn: Giải hệ phương trình trong chương trình toán THPT, HOT
Luận văn: Giải hệ phương trình trong chương trình toán THPT, HOT
Dịch Vụ Viết Bài Trọn Gói ZALO 0917193864
 
Dạng Legendre và ứng dụng.pdf
Dạng Legendre và ứng dụng.pdfDạng Legendre và ứng dụng.pdf
Dạng Legendre và ứng dụng.pdf
NuioKila
 
Đề tài: Phương trình vi phân với toán tử khả nghịch phải, HOT, 9đ
Đề tài: Phương trình vi phân với toán tử khả nghịch phải, HOT, 9đĐề tài: Phương trình vi phân với toán tử khả nghịch phải, HOT, 9đ
Đề tài: Phương trình vi phân với toán tử khả nghịch phải, HOT, 9đ
Dịch vụ viết bài trọn gói ZALO 0917193864
 

Similar to Một số dạng toán về đa thức qua các kỳ thi Olympic 6732069.pdf (20)

Luận văn: Đa tạp tâm của hệ tam phân mũ không đều, HOT
Luận văn: Đa tạp tâm của hệ tam phân mũ không đều, HOTLuận văn: Đa tạp tâm của hệ tam phân mũ không đều, HOT
Luận văn: Đa tạp tâm của hệ tam phân mũ không đều, HOT
 
Luận văn: Đa tạp tâm của hệ tam phân mũ không đều, HAY, 9đ
Luận văn: Đa tạp tâm của hệ tam phân mũ không đều, HAY, 9đLuận văn: Đa tạp tâm của hệ tam phân mũ không đều, HAY, 9đ
Luận văn: Đa tạp tâm của hệ tam phân mũ không đều, HAY, 9đ
 
Đề tài: Tính ổn định của lớp phương trình hàm với cặp biến tự do
Đề tài: Tính ổn định của lớp phương trình hàm với cặp biến tự doĐề tài: Tính ổn định của lớp phương trình hàm với cặp biến tự do
Đề tài: Tính ổn định của lớp phương trình hàm với cặp biến tự do
 
Bat đang thức trong so hoc và m t so Dạng toán liên quan.docx
Bat đang thức trong so hoc và m t so Dạng toán liên quan.docxBat đang thức trong so hoc và m t so Dạng toán liên quan.docx
Bat đang thức trong so hoc và m t so Dạng toán liên quan.docx
 
Luận văn: Lớp bài toán tìm giá trị lớn nhất giá trị nhỏ nhất, HOT
Luận văn: Lớp bài toán tìm giá trị lớn nhất giá trị nhỏ nhất, HOTLuận văn: Lớp bài toán tìm giá trị lớn nhất giá trị nhỏ nhất, HOT
Luận văn: Lớp bài toán tìm giá trị lớn nhất giá trị nhỏ nhất, HOT
 
Đề tài: Phương pháp giải hệ phương trình và hệ bất phương trình
Đề tài: Phương pháp giải hệ phương trình và hệ bất phương trìnhĐề tài: Phương pháp giải hệ phương trình và hệ bất phương trình
Đề tài: Phương pháp giải hệ phương trình và hệ bất phương trình
 
Luận văn: Phương trình phi tuyến không chỉnh loại J - đơn điệu, 9đ
Luận văn: Phương trình phi tuyến không chỉnh loại J - đơn điệu, 9đLuận văn: Phương trình phi tuyến không chỉnh loại J - đơn điệu, 9đ
Luận văn: Phương trình phi tuyến không chỉnh loại J - đơn điệu, 9đ
 
SH_Lien_ND_Dinh ly thang du Trung Hoa_VP_2016_08_16.pdf
SH_Lien_ND_Dinh ly thang du Trung Hoa_VP_2016_08_16.pdfSH_Lien_ND_Dinh ly thang du Trung Hoa_VP_2016_08_16.pdf
SH_Lien_ND_Dinh ly thang du Trung Hoa_VP_2016_08_16.pdf
 
Định lý zsigmondy và Tính chất số học của đa thức.docx
Định lý zsigmondy và Tính chất số học của đa thức.docxĐịnh lý zsigmondy và Tính chất số học của đa thức.docx
Định lý zsigmondy và Tính chất số học của đa thức.docx
 
Bat Đang Thức Và Bài Toán Cực Tr± Trong L P Các Đa Thức Và Phân Thức H So Ngu...
Bat Đang Thức Và Bài Toán Cực Tr± Trong L P Các Đa Thức Và Phân Thức H So Ngu...Bat Đang Thức Và Bài Toán Cực Tr± Trong L P Các Đa Thức Và Phân Thức H So Ngu...
Bat Đang Thức Và Bài Toán Cực Tr± Trong L P Các Đa Thức Và Phân Thức H So Ngu...
 
Một số phương pháp tìm cực trị của các hàm phân thức Sinh bởi số tự nhiên.docx
Một số phương pháp tìm cực trị của các hàm phân thức Sinh bởi số tự nhiên.docxMột số phương pháp tìm cực trị của các hàm phân thức Sinh bởi số tự nhiên.docx
Một số phương pháp tìm cực trị của các hàm phân thức Sinh bởi số tự nhiên.docx
 
Phương Pháp Lư Ng Giác Giải Phương Trình Đa Thức Và M T So Dạng Toán.docx
Phương Pháp Lư Ng Giác Giải Phương Trình Đa Thức Và M T So Dạng Toán.docxPhương Pháp Lư Ng Giác Giải Phương Trình Đa Thức Và M T So Dạng Toán.docx
Phương Pháp Lư Ng Giác Giải Phương Trình Đa Thức Và M T So Dạng Toán.docx
 
Luận văn: Bài toán cực trị với đa thức đối xứng ba biến, HAY - Gửi miễn phí q...
Luận văn: Bài toán cực trị với đa thức đối xứng ba biến, HAY - Gửi miễn phí q...Luận văn: Bài toán cực trị với đa thức đối xứng ba biến, HAY - Gửi miễn phí q...
Luận văn: Bài toán cực trị với đa thức đối xứng ba biến, HAY - Gửi miễn phí q...
 
Đề tài: Lớp bất đẳng thức, bài toán cực trị với đa thức đối xứng, 9đ
Đề tài: Lớp bất đẳng thức, bài toán cực trị với đa thức đối xứng, 9đĐề tài: Lớp bất đẳng thức, bài toán cực trị với đa thức đối xứng, 9đ
Đề tài: Lớp bất đẳng thức, bài toán cực trị với đa thức đối xứng, 9đ
 
Luận văn: Giải hình thức các phương trình tích phân Volterra, HAY
Luận văn: Giải hình thức các phương trình tích phân Volterra, HAYLuận văn: Giải hình thức các phương trình tích phân Volterra, HAY
Luận văn: Giải hình thức các phương trình tích phân Volterra, HAY
 
Bài giảng chi tiết giải tích 1 07 2013 bo môn toán
Bài giảng chi tiết giải tích 1  07 2013 bo môn toánBài giảng chi tiết giải tích 1  07 2013 bo môn toán
Bài giảng chi tiết giải tích 1 07 2013 bo môn toán
 
Luận văn: Phương pháp giải hệ phương trình trong toán THPT
Luận văn: Phương pháp giải hệ phương trình trong toán THPTLuận văn: Phương pháp giải hệ phương trình trong toán THPT
Luận văn: Phương pháp giải hệ phương trình trong toán THPT
 
Luận văn: Giải hệ phương trình trong chương trình toán THPT, HOT
Luận văn: Giải hệ phương trình trong chương trình toán THPT, HOTLuận văn: Giải hệ phương trình trong chương trình toán THPT, HOT
Luận văn: Giải hệ phương trình trong chương trình toán THPT, HOT
 
Dạng Legendre và ứng dụng.pdf
Dạng Legendre và ứng dụng.pdfDạng Legendre và ứng dụng.pdf
Dạng Legendre và ứng dụng.pdf
 
Đề tài: Phương trình vi phân với toán tử khả nghịch phải, HOT, 9đ
Đề tài: Phương trình vi phân với toán tử khả nghịch phải, HOT, 9đĐề tài: Phương trình vi phân với toán tử khả nghịch phải, HOT, 9đ
Đề tài: Phương trình vi phân với toán tử khả nghịch phải, HOT, 9đ
 

More from TieuNgocLy

THI HÀNH ÁN HÌNH SỰ TỪ THỰC TIỄN TỈNH PHÚ THỌ.pdf
THI HÀNH ÁN HÌNH SỰ TỪ THỰC TIỄN TỈNH PHÚ THỌ.pdfTHI HÀNH ÁN HÌNH SỰ TỪ THỰC TIỄN TỈNH PHÚ THỌ.pdf
THI HÀNH ÁN HÌNH SỰ TỪ THỰC TIỄN TỈNH PHÚ THỌ.pdf
TieuNgocLy
 
Cách trưng bày và bố trí sản phẩm của circle k tại Việt Nam 9870993.pdf
Cách trưng bày và bố trí sản phẩm của circle k tại Việt Nam 9870993.pdfCách trưng bày và bố trí sản phẩm của circle k tại Việt Nam 9870993.pdf
Cách trưng bày và bố trí sản phẩm của circle k tại Việt Nam 9870993.pdf
TieuNgocLy
 
HẠ THÂN NHIỆT ĐIỀU TRỊ TRONG NGỪNG TUẦN HOÀN- THỰC TẾ TẠI VIỆT NAM.pdf
HẠ THÂN NHIỆT ĐIỀU TRỊ TRONG NGỪNG TUẦN HOÀN- THỰC TẾ TẠI VIỆT NAM.pdfHẠ THÂN NHIỆT ĐIỀU TRỊ TRONG NGỪNG TUẦN HOÀN- THỰC TẾ TẠI VIỆT NAM.pdf
HẠ THÂN NHIỆT ĐIỀU TRỊ TRONG NGỪNG TUẦN HOÀN- THỰC TẾ TẠI VIỆT NAM.pdf
TieuNgocLy
 
BÁO CHÍ VỚI VẤN ĐỀ “GIẢI CỨU NÔNG SẢN” CHO NÔNG DÂN - Luận văn Thạc sĩ chuyên...
BÁO CHÍ VỚI VẤN ĐỀ “GIẢI CỨU NÔNG SẢN” CHO NÔNG DÂN - Luận văn Thạc sĩ chuyên...BÁO CHÍ VỚI VẤN ĐỀ “GIẢI CỨU NÔNG SẢN” CHO NÔNG DÂN - Luận văn Thạc sĩ chuyên...
BÁO CHÍ VỚI VẤN ĐỀ “GIẢI CỨU NÔNG SẢN” CHO NÔNG DÂN - Luận văn Thạc sĩ chuyên...
TieuNgocLy
 
现代汉语广告中的成语研究 = Nghiên cứu thành ngữ trong ngôn ngữ quảng cáo của tiếng Hán hi...
现代汉语广告中的成语研究 = Nghiên cứu thành ngữ trong ngôn ngữ quảng cáo của tiếng Hán hi...现代汉语广告中的成语研究 = Nghiên cứu thành ngữ trong ngôn ngữ quảng cáo của tiếng Hán hi...
现代汉语广告中的成语研究 = Nghiên cứu thành ngữ trong ngôn ngữ quảng cáo của tiếng Hán hi...
TieuNgocLy
 
Nghiên cứu hệ thống chống bó cứng phanh trên xe mazda CX 5 2013.pdf
Nghiên cứu hệ thống chống bó cứng phanh trên xe mazda CX 5 2013.pdfNghiên cứu hệ thống chống bó cứng phanh trên xe mazda CX 5 2013.pdf
Nghiên cứu hệ thống chống bó cứng phanh trên xe mazda CX 5 2013.pdf
TieuNgocLy
 
Chức Năng Hoạch Định Quản Trị Học.pdf
Chức Năng Hoạch Định Quản Trị Học.pdfChức Năng Hoạch Định Quản Trị Học.pdf
Chức Năng Hoạch Định Quản Trị Học.pdf
TieuNgocLy
 
NHẬN THỨC VỀ YẾU TỐ NGUY CƠ VÀ BIỂU HIỆN CẢNH BÁO ĐỘT QỤY NÃO CỦA NGƯỜI BỆNH ...
NHẬN THỨC VỀ YẾU TỐ NGUY CƠ VÀ BIỂU HIỆN CẢNH BÁO ĐỘT QỤY NÃO CỦA NGƯỜI BỆNH ...NHẬN THỨC VỀ YẾU TỐ NGUY CƠ VÀ BIỂU HIỆN CẢNH BÁO ĐỘT QỤY NÃO CỦA NGƯỜI BỆNH ...
NHẬN THỨC VỀ YẾU TỐ NGUY CƠ VÀ BIỂU HIỆN CẢNH BÁO ĐỘT QỤY NÃO CỦA NGƯỜI BỆNH ...
TieuNgocLy
 
HỘI THẢO CƠ CHẾ CHÍNH SÁCH CUNG ỨNG DỊCH VỤ CÔNG ÍCH TẠI CÁC ĐÔ THỊ Ở VIỆT NA...
HỘI THẢO CƠ CHẾ CHÍNH SÁCH CUNG ỨNG DỊCH VỤ CÔNG ÍCH TẠI CÁC ĐÔ THỊ Ở VIỆT NA...HỘI THẢO CƠ CHẾ CHÍNH SÁCH CUNG ỨNG DỊCH VỤ CÔNG ÍCH TẠI CÁC ĐÔ THỊ Ở VIỆT NA...
HỘI THẢO CƠ CHẾ CHÍNH SÁCH CUNG ỨNG DỊCH VỤ CÔNG ÍCH TẠI CÁC ĐÔ THỊ Ở VIỆT NA...
TieuNgocLy
 
Nghiên cứu quá trình thụ đắc từ li hợp trong tiếng Hán hiện đại của sinh viên...
Nghiên cứu quá trình thụ đắc từ li hợp trong tiếng Hán hiện đại của sinh viên...Nghiên cứu quá trình thụ đắc từ li hợp trong tiếng Hán hiện đại của sinh viên...
Nghiên cứu quá trình thụ đắc từ li hợp trong tiếng Hán hiện đại của sinh viên...
TieuNgocLy
 
Báo Cáo Thực Tập Tốt Nghiệp Thông Tin Vô Tuyến, Chuyển Mạch Và Thông Tin Quan...
Báo Cáo Thực Tập Tốt Nghiệp Thông Tin Vô Tuyến, Chuyển Mạch Và Thông Tin Quan...Báo Cáo Thực Tập Tốt Nghiệp Thông Tin Vô Tuyến, Chuyển Mạch Và Thông Tin Quan...
Báo Cáo Thực Tập Tốt Nghiệp Thông Tin Vô Tuyến, Chuyển Mạch Và Thông Tin Quan...
TieuNgocLy
 
HIỆP ĐỊNH THÀNH LẬP KHU VỰC THƯƠNG MẠI TỰ DO ASEAN – ÚC – NIU DILÂN (AANZFTA)...
HIỆP ĐỊNH THÀNH LẬP KHU VỰC THƯƠNG MẠI TỰ DO ASEAN – ÚC – NIU DILÂN (AANZFTA)...HIỆP ĐỊNH THÀNH LẬP KHU VỰC THƯƠNG MẠI TỰ DO ASEAN – ÚC – NIU DILÂN (AANZFTA)...
HIỆP ĐỊNH THÀNH LẬP KHU VỰC THƯƠNG MẠI TỰ DO ASEAN – ÚC – NIU DILÂN (AANZFTA)...
TieuNgocLy
 
Những vấn đề pháp lý về chống bán phá giá hàng hóa nhập khẩu vào Việt Nam.pdf
Những vấn đề pháp lý về chống bán phá giá hàng hóa nhập khẩu vào Việt Nam.pdfNhững vấn đề pháp lý về chống bán phá giá hàng hóa nhập khẩu vào Việt Nam.pdf
Những vấn đề pháp lý về chống bán phá giá hàng hóa nhập khẩu vào Việt Nam.pdf
TieuNgocLy
 
Pháp luật về quản lý chất thải nguy hại trong khu công nghiệp ở Việt Nam.pdf
Pháp luật về quản lý chất thải nguy hại trong khu công nghiệp ở Việt Nam.pdfPháp luật về quản lý chất thải nguy hại trong khu công nghiệp ở Việt Nam.pdf
Pháp luật về quản lý chất thải nguy hại trong khu công nghiệp ở Việt Nam.pdf
TieuNgocLy
 
Thiết Kế Hệ Thống Cung Cấp Điện Cho Tòa Nhà Cao Tầng Có Ứng Dụng Các Phương P...
Thiết Kế Hệ Thống Cung Cấp Điện Cho Tòa Nhà Cao Tầng Có Ứng Dụng Các Phương P...Thiết Kế Hệ Thống Cung Cấp Điện Cho Tòa Nhà Cao Tầng Có Ứng Dụng Các Phương P...
Thiết Kế Hệ Thống Cung Cấp Điện Cho Tòa Nhà Cao Tầng Có Ứng Dụng Các Phương P...
TieuNgocLy
 
Bài Giảng Chứng Khoán Phái Sinh.pdf
Bài Giảng Chứng Khoán Phái Sinh.pdfBài Giảng Chứng Khoán Phái Sinh.pdf
Bài Giảng Chứng Khoán Phái Sinh.pdf
TieuNgocLy
 
Hội Thảo, Tập Huấn, Rút Kinh Nghiệm Dạy Học Theo Mô Hình Trường Học Mới Việt ...
Hội Thảo, Tập Huấn, Rút Kinh Nghiệm Dạy Học Theo Mô Hình Trường Học Mới Việt ...Hội Thảo, Tập Huấn, Rút Kinh Nghiệm Dạy Học Theo Mô Hình Trường Học Mới Việt ...
Hội Thảo, Tập Huấn, Rút Kinh Nghiệm Dạy Học Theo Mô Hình Trường Học Mới Việt ...
TieuNgocLy
 
Intangible Values in Financial Accounting and Reporting An Analysis from the ...
Intangible Values in Financial Accounting and Reporting An Analysis from the ...Intangible Values in Financial Accounting and Reporting An Analysis from the ...
Intangible Values in Financial Accounting and Reporting An Analysis from the ...
TieuNgocLy
 
Bài Giảng Các Phương Pháp Dạy Học Hiện Đại.pdf
Bài Giảng Các Phương Pháp Dạy Học Hiện Đại.pdfBài Giảng Các Phương Pháp Dạy Học Hiện Đại.pdf
Bài Giảng Các Phương Pháp Dạy Học Hiện Đại.pdf
TieuNgocLy
 
Những Kiến Thức Cơ Bản Của Tâm Lý Học Lứa Tuổi Và Tâm Lý Học Sư Phạm.pdf
Những Kiến Thức Cơ Bản Của Tâm Lý Học Lứa Tuổi Và Tâm Lý Học Sư Phạm.pdfNhững Kiến Thức Cơ Bản Của Tâm Lý Học Lứa Tuổi Và Tâm Lý Học Sư Phạm.pdf
Những Kiến Thức Cơ Bản Của Tâm Lý Học Lứa Tuổi Và Tâm Lý Học Sư Phạm.pdf
TieuNgocLy
 

More from TieuNgocLy (20)

THI HÀNH ÁN HÌNH SỰ TỪ THỰC TIỄN TỈNH PHÚ THỌ.pdf
THI HÀNH ÁN HÌNH SỰ TỪ THỰC TIỄN TỈNH PHÚ THỌ.pdfTHI HÀNH ÁN HÌNH SỰ TỪ THỰC TIỄN TỈNH PHÚ THỌ.pdf
THI HÀNH ÁN HÌNH SỰ TỪ THỰC TIỄN TỈNH PHÚ THỌ.pdf
 
Cách trưng bày và bố trí sản phẩm của circle k tại Việt Nam 9870993.pdf
Cách trưng bày và bố trí sản phẩm của circle k tại Việt Nam 9870993.pdfCách trưng bày và bố trí sản phẩm của circle k tại Việt Nam 9870993.pdf
Cách trưng bày và bố trí sản phẩm của circle k tại Việt Nam 9870993.pdf
 
HẠ THÂN NHIỆT ĐIỀU TRỊ TRONG NGỪNG TUẦN HOÀN- THỰC TẾ TẠI VIỆT NAM.pdf
HẠ THÂN NHIỆT ĐIỀU TRỊ TRONG NGỪNG TUẦN HOÀN- THỰC TẾ TẠI VIỆT NAM.pdfHẠ THÂN NHIỆT ĐIỀU TRỊ TRONG NGỪNG TUẦN HOÀN- THỰC TẾ TẠI VIỆT NAM.pdf
HẠ THÂN NHIỆT ĐIỀU TRỊ TRONG NGỪNG TUẦN HOÀN- THỰC TẾ TẠI VIỆT NAM.pdf
 
BÁO CHÍ VỚI VẤN ĐỀ “GIẢI CỨU NÔNG SẢN” CHO NÔNG DÂN - Luận văn Thạc sĩ chuyên...
BÁO CHÍ VỚI VẤN ĐỀ “GIẢI CỨU NÔNG SẢN” CHO NÔNG DÂN - Luận văn Thạc sĩ chuyên...BÁO CHÍ VỚI VẤN ĐỀ “GIẢI CỨU NÔNG SẢN” CHO NÔNG DÂN - Luận văn Thạc sĩ chuyên...
BÁO CHÍ VỚI VẤN ĐỀ “GIẢI CỨU NÔNG SẢN” CHO NÔNG DÂN - Luận văn Thạc sĩ chuyên...
 
现代汉语广告中的成语研究 = Nghiên cứu thành ngữ trong ngôn ngữ quảng cáo của tiếng Hán hi...
现代汉语广告中的成语研究 = Nghiên cứu thành ngữ trong ngôn ngữ quảng cáo của tiếng Hán hi...现代汉语广告中的成语研究 = Nghiên cứu thành ngữ trong ngôn ngữ quảng cáo của tiếng Hán hi...
现代汉语广告中的成语研究 = Nghiên cứu thành ngữ trong ngôn ngữ quảng cáo của tiếng Hán hi...
 
Nghiên cứu hệ thống chống bó cứng phanh trên xe mazda CX 5 2013.pdf
Nghiên cứu hệ thống chống bó cứng phanh trên xe mazda CX 5 2013.pdfNghiên cứu hệ thống chống bó cứng phanh trên xe mazda CX 5 2013.pdf
Nghiên cứu hệ thống chống bó cứng phanh trên xe mazda CX 5 2013.pdf
 
Chức Năng Hoạch Định Quản Trị Học.pdf
Chức Năng Hoạch Định Quản Trị Học.pdfChức Năng Hoạch Định Quản Trị Học.pdf
Chức Năng Hoạch Định Quản Trị Học.pdf
 
NHẬN THỨC VỀ YẾU TỐ NGUY CƠ VÀ BIỂU HIỆN CẢNH BÁO ĐỘT QỤY NÃO CỦA NGƯỜI BỆNH ...
NHẬN THỨC VỀ YẾU TỐ NGUY CƠ VÀ BIỂU HIỆN CẢNH BÁO ĐỘT QỤY NÃO CỦA NGƯỜI BỆNH ...NHẬN THỨC VỀ YẾU TỐ NGUY CƠ VÀ BIỂU HIỆN CẢNH BÁO ĐỘT QỤY NÃO CỦA NGƯỜI BỆNH ...
NHẬN THỨC VỀ YẾU TỐ NGUY CƠ VÀ BIỂU HIỆN CẢNH BÁO ĐỘT QỤY NÃO CỦA NGƯỜI BỆNH ...
 
HỘI THẢO CƠ CHẾ CHÍNH SÁCH CUNG ỨNG DỊCH VỤ CÔNG ÍCH TẠI CÁC ĐÔ THỊ Ở VIỆT NA...
HỘI THẢO CƠ CHẾ CHÍNH SÁCH CUNG ỨNG DỊCH VỤ CÔNG ÍCH TẠI CÁC ĐÔ THỊ Ở VIỆT NA...HỘI THẢO CƠ CHẾ CHÍNH SÁCH CUNG ỨNG DỊCH VỤ CÔNG ÍCH TẠI CÁC ĐÔ THỊ Ở VIỆT NA...
HỘI THẢO CƠ CHẾ CHÍNH SÁCH CUNG ỨNG DỊCH VỤ CÔNG ÍCH TẠI CÁC ĐÔ THỊ Ở VIỆT NA...
 
Nghiên cứu quá trình thụ đắc từ li hợp trong tiếng Hán hiện đại của sinh viên...
Nghiên cứu quá trình thụ đắc từ li hợp trong tiếng Hán hiện đại của sinh viên...Nghiên cứu quá trình thụ đắc từ li hợp trong tiếng Hán hiện đại của sinh viên...
Nghiên cứu quá trình thụ đắc từ li hợp trong tiếng Hán hiện đại của sinh viên...
 
Báo Cáo Thực Tập Tốt Nghiệp Thông Tin Vô Tuyến, Chuyển Mạch Và Thông Tin Quan...
Báo Cáo Thực Tập Tốt Nghiệp Thông Tin Vô Tuyến, Chuyển Mạch Và Thông Tin Quan...Báo Cáo Thực Tập Tốt Nghiệp Thông Tin Vô Tuyến, Chuyển Mạch Và Thông Tin Quan...
Báo Cáo Thực Tập Tốt Nghiệp Thông Tin Vô Tuyến, Chuyển Mạch Và Thông Tin Quan...
 
HIỆP ĐỊNH THÀNH LẬP KHU VỰC THƯƠNG MẠI TỰ DO ASEAN – ÚC – NIU DILÂN (AANZFTA)...
HIỆP ĐỊNH THÀNH LẬP KHU VỰC THƯƠNG MẠI TỰ DO ASEAN – ÚC – NIU DILÂN (AANZFTA)...HIỆP ĐỊNH THÀNH LẬP KHU VỰC THƯƠNG MẠI TỰ DO ASEAN – ÚC – NIU DILÂN (AANZFTA)...
HIỆP ĐỊNH THÀNH LẬP KHU VỰC THƯƠNG MẠI TỰ DO ASEAN – ÚC – NIU DILÂN (AANZFTA)...
 
Những vấn đề pháp lý về chống bán phá giá hàng hóa nhập khẩu vào Việt Nam.pdf
Những vấn đề pháp lý về chống bán phá giá hàng hóa nhập khẩu vào Việt Nam.pdfNhững vấn đề pháp lý về chống bán phá giá hàng hóa nhập khẩu vào Việt Nam.pdf
Những vấn đề pháp lý về chống bán phá giá hàng hóa nhập khẩu vào Việt Nam.pdf
 
Pháp luật về quản lý chất thải nguy hại trong khu công nghiệp ở Việt Nam.pdf
Pháp luật về quản lý chất thải nguy hại trong khu công nghiệp ở Việt Nam.pdfPháp luật về quản lý chất thải nguy hại trong khu công nghiệp ở Việt Nam.pdf
Pháp luật về quản lý chất thải nguy hại trong khu công nghiệp ở Việt Nam.pdf
 
Thiết Kế Hệ Thống Cung Cấp Điện Cho Tòa Nhà Cao Tầng Có Ứng Dụng Các Phương P...
Thiết Kế Hệ Thống Cung Cấp Điện Cho Tòa Nhà Cao Tầng Có Ứng Dụng Các Phương P...Thiết Kế Hệ Thống Cung Cấp Điện Cho Tòa Nhà Cao Tầng Có Ứng Dụng Các Phương P...
Thiết Kế Hệ Thống Cung Cấp Điện Cho Tòa Nhà Cao Tầng Có Ứng Dụng Các Phương P...
 
Bài Giảng Chứng Khoán Phái Sinh.pdf
Bài Giảng Chứng Khoán Phái Sinh.pdfBài Giảng Chứng Khoán Phái Sinh.pdf
Bài Giảng Chứng Khoán Phái Sinh.pdf
 
Hội Thảo, Tập Huấn, Rút Kinh Nghiệm Dạy Học Theo Mô Hình Trường Học Mới Việt ...
Hội Thảo, Tập Huấn, Rút Kinh Nghiệm Dạy Học Theo Mô Hình Trường Học Mới Việt ...Hội Thảo, Tập Huấn, Rút Kinh Nghiệm Dạy Học Theo Mô Hình Trường Học Mới Việt ...
Hội Thảo, Tập Huấn, Rút Kinh Nghiệm Dạy Học Theo Mô Hình Trường Học Mới Việt ...
 
Intangible Values in Financial Accounting and Reporting An Analysis from the ...
Intangible Values in Financial Accounting and Reporting An Analysis from the ...Intangible Values in Financial Accounting and Reporting An Analysis from the ...
Intangible Values in Financial Accounting and Reporting An Analysis from the ...
 
Bài Giảng Các Phương Pháp Dạy Học Hiện Đại.pdf
Bài Giảng Các Phương Pháp Dạy Học Hiện Đại.pdfBài Giảng Các Phương Pháp Dạy Học Hiện Đại.pdf
Bài Giảng Các Phương Pháp Dạy Học Hiện Đại.pdf
 
Những Kiến Thức Cơ Bản Của Tâm Lý Học Lứa Tuổi Và Tâm Lý Học Sư Phạm.pdf
Những Kiến Thức Cơ Bản Của Tâm Lý Học Lứa Tuổi Và Tâm Lý Học Sư Phạm.pdfNhững Kiến Thức Cơ Bản Của Tâm Lý Học Lứa Tuổi Và Tâm Lý Học Sư Phạm.pdf
Những Kiến Thức Cơ Bản Của Tâm Lý Học Lứa Tuổi Và Tâm Lý Học Sư Phạm.pdf
 

Recently uploaded

Ảnh hưởng của nhân sinh quan Phật giáo đến đời sống tinh thần Việt Nam hiện nay
Ảnh hưởng của nhân sinh quan Phật giáo đến đời sống tinh thần Việt Nam hiện nayẢnh hưởng của nhân sinh quan Phật giáo đến đời sống tinh thần Việt Nam hiện nay
Ảnh hưởng của nhân sinh quan Phật giáo đến đời sống tinh thần Việt Nam hiện nay
chinhkt50
 
DS thi KTHP HK2 (dot 3) nam hoc 2023-2024.pdf
DS thi KTHP HK2 (dot 3) nam hoc 2023-2024.pdfDS thi KTHP HK2 (dot 3) nam hoc 2023-2024.pdf
DS thi KTHP HK2 (dot 3) nam hoc 2023-2024.pdf
thanhluan21
 
30 - ĐỀ THI HSG - HÓA HỌC 9 - NĂM HỌC 2021 - 2022.pdf
30 - ĐỀ THI HSG - HÓA HỌC 9 - NĂM HỌC 2021 - 2022.pdf30 - ĐỀ THI HSG - HÓA HỌC 9 - NĂM HỌC 2021 - 2022.pdf
30 - ĐỀ THI HSG - HÓA HỌC 9 - NĂM HỌC 2021 - 2022.pdf
ngocnguyensp1
 
BAI TAP ON HE LOP 2 LEN 3 MON TIENG VIET.pdf
BAI TAP ON HE LOP 2 LEN 3 MON TIENG VIET.pdfBAI TAP ON HE LOP 2 LEN 3 MON TIENG VIET.pdf
BAI TAP ON HE LOP 2 LEN 3 MON TIENG VIET.pdf
phamthuhoai20102005
 
Khoá luận tốt nghiệp ngành Truyền thông đa phương tiện Xây dựng kế hoạch truy...
Khoá luận tốt nghiệp ngành Truyền thông đa phương tiện Xây dựng kế hoạch truy...Khoá luận tốt nghiệp ngành Truyền thông đa phương tiện Xây dựng kế hoạch truy...
Khoá luận tốt nghiệp ngành Truyền thông đa phương tiện Xây dựng kế hoạch truy...
https://www.facebook.com/garmentspace
 
CÁC BIỆN PHÁP KỸ THUẬT AN TOÀN KHI XÃY RA HỎA HOẠN TRONG.pptx
CÁC BIỆN PHÁP KỸ THUẬT AN TOÀN KHI XÃY RA HỎA HOẠN TRONG.pptxCÁC BIỆN PHÁP KỸ THUẬT AN TOÀN KHI XÃY RA HỎA HOẠN TRONG.pptx
CÁC BIỆN PHÁP KỸ THUẬT AN TOÀN KHI XÃY RA HỎA HOẠN TRONG.pptx
CNGTRC3
 
Nghiên cứu cơ chế và động học phản ứng giữa hợp chất Aniline (C6H5NH2) với gố...
Nghiên cứu cơ chế và động học phản ứng giữa hợp chất Aniline (C6H5NH2) với gố...Nghiên cứu cơ chế và động học phản ứng giữa hợp chất Aniline (C6H5NH2) với gố...
Nghiên cứu cơ chế và động học phản ứng giữa hợp chất Aniline (C6H5NH2) với gố...
Nguyen Thanh Tu Collection
 
98 BÀI LUYỆN NGHE TUYỂN SINH VÀO LỚP 10 TIẾNG ANH DẠNG TRẮC NGHIỆM 4 CÂU TRẢ ...
98 BÀI LUYỆN NGHE TUYỂN SINH VÀO LỚP 10 TIẾNG ANH DẠNG TRẮC NGHIỆM 4 CÂU TRẢ ...98 BÀI LUYỆN NGHE TUYỂN SINH VÀO LỚP 10 TIẾNG ANH DẠNG TRẮC NGHIỆM 4 CÂU TRẢ ...
98 BÀI LUYỆN NGHE TUYỂN SINH VÀO LỚP 10 TIẾNG ANH DẠNG TRẮC NGHIỆM 4 CÂU TRẢ ...
Nguyen Thanh Tu Collection
 
GIÁO TRÌNH 2-TÀI LIỆU SỬA CHỮA BOARD MONO TỦ LẠNH MÁY GIẶT ĐIỀU HÒA.pdf
GIÁO TRÌNH 2-TÀI LIỆU SỬA CHỮA BOARD MONO TỦ LẠNH MÁY GIẶT ĐIỀU HÒA.pdfGIÁO TRÌNH 2-TÀI LIỆU SỬA CHỮA BOARD MONO TỦ LẠNH MÁY GIẶT ĐIỀU HÒA.pdf
GIÁO TRÌNH 2-TÀI LIỆU SỬA CHỮA BOARD MONO TỦ LẠNH MÁY GIẶT ĐIỀU HÒA.pdf
Điện Lạnh Bách Khoa Hà Nội
 
CHUYÊN ĐỀ BỒI DƯỠNG HỌC SINH GIỎI KHOA HỌC TỰ NHIÊN 9 CHƯƠNG TRÌNH MỚI - PHẦN...
CHUYÊN ĐỀ BỒI DƯỠNG HỌC SINH GIỎI KHOA HỌC TỰ NHIÊN 9 CHƯƠNG TRÌNH MỚI - PHẦN...CHUYÊN ĐỀ BỒI DƯỠNG HỌC SINH GIỎI KHOA HỌC TỰ NHIÊN 9 CHƯƠNG TRÌNH MỚI - PHẦN...
CHUYÊN ĐỀ BỒI DƯỠNG HỌC SINH GIỎI KHOA HỌC TỰ NHIÊN 9 CHƯƠNG TRÌNH MỚI - PHẦN...
Nguyen Thanh Tu Collection
 
GIAO TRINH TRIET HOC MAC - LENIN (Quoc gia).pdf
GIAO TRINH TRIET HOC MAC - LENIN (Quoc gia).pdfGIAO TRINH TRIET HOC MAC - LENIN (Quoc gia).pdf
GIAO TRINH TRIET HOC MAC - LENIN (Quoc gia).pdf
LngHu10
 

Recently uploaded (11)

Ảnh hưởng của nhân sinh quan Phật giáo đến đời sống tinh thần Việt Nam hiện nay
Ảnh hưởng của nhân sinh quan Phật giáo đến đời sống tinh thần Việt Nam hiện nayẢnh hưởng của nhân sinh quan Phật giáo đến đời sống tinh thần Việt Nam hiện nay
Ảnh hưởng của nhân sinh quan Phật giáo đến đời sống tinh thần Việt Nam hiện nay
 
DS thi KTHP HK2 (dot 3) nam hoc 2023-2024.pdf
DS thi KTHP HK2 (dot 3) nam hoc 2023-2024.pdfDS thi KTHP HK2 (dot 3) nam hoc 2023-2024.pdf
DS thi KTHP HK2 (dot 3) nam hoc 2023-2024.pdf
 
30 - ĐỀ THI HSG - HÓA HỌC 9 - NĂM HỌC 2021 - 2022.pdf
30 - ĐỀ THI HSG - HÓA HỌC 9 - NĂM HỌC 2021 - 2022.pdf30 - ĐỀ THI HSG - HÓA HỌC 9 - NĂM HỌC 2021 - 2022.pdf
30 - ĐỀ THI HSG - HÓA HỌC 9 - NĂM HỌC 2021 - 2022.pdf
 
BAI TAP ON HE LOP 2 LEN 3 MON TIENG VIET.pdf
BAI TAP ON HE LOP 2 LEN 3 MON TIENG VIET.pdfBAI TAP ON HE LOP 2 LEN 3 MON TIENG VIET.pdf
BAI TAP ON HE LOP 2 LEN 3 MON TIENG VIET.pdf
 
Khoá luận tốt nghiệp ngành Truyền thông đa phương tiện Xây dựng kế hoạch truy...
Khoá luận tốt nghiệp ngành Truyền thông đa phương tiện Xây dựng kế hoạch truy...Khoá luận tốt nghiệp ngành Truyền thông đa phương tiện Xây dựng kế hoạch truy...
Khoá luận tốt nghiệp ngành Truyền thông đa phương tiện Xây dựng kế hoạch truy...
 
CÁC BIỆN PHÁP KỸ THUẬT AN TOÀN KHI XÃY RA HỎA HOẠN TRONG.pptx
CÁC BIỆN PHÁP KỸ THUẬT AN TOÀN KHI XÃY RA HỎA HOẠN TRONG.pptxCÁC BIỆN PHÁP KỸ THUẬT AN TOÀN KHI XÃY RA HỎA HOẠN TRONG.pptx
CÁC BIỆN PHÁP KỸ THUẬT AN TOÀN KHI XÃY RA HỎA HOẠN TRONG.pptx
 
Nghiên cứu cơ chế và động học phản ứng giữa hợp chất Aniline (C6H5NH2) với gố...
Nghiên cứu cơ chế và động học phản ứng giữa hợp chất Aniline (C6H5NH2) với gố...Nghiên cứu cơ chế và động học phản ứng giữa hợp chất Aniline (C6H5NH2) với gố...
Nghiên cứu cơ chế và động học phản ứng giữa hợp chất Aniline (C6H5NH2) với gố...
 
98 BÀI LUYỆN NGHE TUYỂN SINH VÀO LỚP 10 TIẾNG ANH DẠNG TRẮC NGHIỆM 4 CÂU TRẢ ...
98 BÀI LUYỆN NGHE TUYỂN SINH VÀO LỚP 10 TIẾNG ANH DẠNG TRẮC NGHIỆM 4 CÂU TRẢ ...98 BÀI LUYỆN NGHE TUYỂN SINH VÀO LỚP 10 TIẾNG ANH DẠNG TRẮC NGHIỆM 4 CÂU TRẢ ...
98 BÀI LUYỆN NGHE TUYỂN SINH VÀO LỚP 10 TIẾNG ANH DẠNG TRẮC NGHIỆM 4 CÂU TRẢ ...
 
GIÁO TRÌNH 2-TÀI LIỆU SỬA CHỮA BOARD MONO TỦ LẠNH MÁY GIẶT ĐIỀU HÒA.pdf
GIÁO TRÌNH 2-TÀI LIỆU SỬA CHỮA BOARD MONO TỦ LẠNH MÁY GIẶT ĐIỀU HÒA.pdfGIÁO TRÌNH 2-TÀI LIỆU SỬA CHỮA BOARD MONO TỦ LẠNH MÁY GIẶT ĐIỀU HÒA.pdf
GIÁO TRÌNH 2-TÀI LIỆU SỬA CHỮA BOARD MONO TỦ LẠNH MÁY GIẶT ĐIỀU HÒA.pdf
 
CHUYÊN ĐỀ BỒI DƯỠNG HỌC SINH GIỎI KHOA HỌC TỰ NHIÊN 9 CHƯƠNG TRÌNH MỚI - PHẦN...
CHUYÊN ĐỀ BỒI DƯỠNG HỌC SINH GIỎI KHOA HỌC TỰ NHIÊN 9 CHƯƠNG TRÌNH MỚI - PHẦN...CHUYÊN ĐỀ BỒI DƯỠNG HỌC SINH GIỎI KHOA HỌC TỰ NHIÊN 9 CHƯƠNG TRÌNH MỚI - PHẦN...
CHUYÊN ĐỀ BỒI DƯỠNG HỌC SINH GIỎI KHOA HỌC TỰ NHIÊN 9 CHƯƠNG TRÌNH MỚI - PHẦN...
 
GIAO TRINH TRIET HOC MAC - LENIN (Quoc gia).pdf
GIAO TRINH TRIET HOC MAC - LENIN (Quoc gia).pdfGIAO TRINH TRIET HOC MAC - LENIN (Quoc gia).pdf
GIAO TRINH TRIET HOC MAC - LENIN (Quoc gia).pdf
 

Một số dạng toán về đa thức qua các kỳ thi Olympic 6732069.pdf

  • 1. ĐẠI HỌC QUỐC GIA HÀ NỘI TRƯỜNG ĐẠI HỌC KHOA HỌC TỰ NHIÊN TRẦN THỊ VIẾT THỦY MỘT SỐ DẠNG TOÁN VỀ ĐA THỨC QUA CÁC KỲ THI OLYMPIC LUẬN VĂN THẠC SỸ TOÁN HỌC HÀ NỘI - NĂM 2017
  • 2. ĐẠI HỌC QUỐC GIA HÀ NỘI TRƯỜNG ĐẠI HỌC KHOA HỌC TỰ NHIÊN TRẦN THỊ VIẾT THỦY MỘT SỐ DẠNG TOÁN VỀ ĐA THỨC QUA CÁC KỲ THI OLYMPIC LUẬN VĂN THẠC SỸ TOÁN HỌC Mã số: 60.46.01.13 Người hướng dẫn khoa học GS. TSKH. NGUYỄN VĂN MẬU HÀ NỘI - NĂM 2017
  • 3. Mục lục Mở đầu 1 1 Xác định đa thức 3 1.1 Một số tính chất cơ bản của đa thức . . . . . . . . . . . . . 3 1.2 Xác định đa thức theo các đặc trưng số học . . . . . . . . . 5 1.3 Xác định đa thức theo các đặc trưng nghiệm . . . . . . . . 13 1.4 Xác dịnh đa thức theo phép biến đổi vi phân hàm . . . . . 19 2 Ước lượng đa thức 28 2.1 Đa thức Chebyshev và các tính chất . . . . . . . . . . . . . 28 2.2 Các dạng toán liên quan đến đa thức Chebyshev . . . . . . 32 2.3 Ước lượng, giá trị cực trị của đa thức . . . . . . . . . . . . 36 3 Một số dạng toán liên quan 47 3.1 Đa thức với hệ số nguyên và đa thức nhận giá trị nguyên . . 47 3.2 Đa thức với hệ số hữu tỷ và phân thức hữu tỷ . . . . . . . . 58 3.3 Ứng dụng tính chất nghiệm của đa thức . . . . . . . . . . . 67 Kết luận 72 Tài liệu tham khảo 74 1
  • 4. MỞ ĐẦU Một chuyên đề cơ bản và quan trọng trong đại số, trong toán học nói chung là chuyên đề đa thức. Đa thức có vị trí quan trọng trong kiến thức toán nói chung, trong chương trình phổ thông, và đặc biệt đối với các lớp chuyên toán nói riêng. Trong các kì thi chọn học sinh giỏi toán, vô địch Quốc gia, Quốc tế và Olympic sinh viên, các dạng toán về đa thức thường xuất hiện với mức độ khó và rất khó. Nhiều đề thi cùng đáp án đã được đăngtải ở tạp chí toán học và tuổi trẻ, ở nhiều sách tham khảo nhưng chưa thật đầy đủ. Với mong muốn có một chuyên đề giúp nâng cao kiến thức về đa thức và bồi dưỡng học sinh giỏi toán, luận văn "Một số dạng toán về đa thức qua các đề thi Olympic” nhằm tìm hiểu, thu thập các tài liệu biên soạn gồm các đề thi học sinh giỏi toán THPT Quốc gia, đề thi toán Quốc tế, đề thi Olympic sinh viên. Các dạng toán về đa thức rất phong phú, đa dạng về thể loại và phương pháp, thường rất rất phức tạp nên khó phân loại và hệ thống thành các chuyên đề riêng biệt .Tuy vậy, để đáp ứng nhu cầu về giảng dạy, học tập, luận văn "Một số dạng toán về đa thức qua các đề thi Olympic” cũng cố gắng tối đa sắp xếp theo trình tự hợp lí nhằm giúp tiếp cận từng bước , từng mức độ kiến thức và luyện tập kĩ năng giải toán. Luận văn được chia làm 3 chương. Chương 1. Xác định và tồn tại đa thức. Chương 2. Ước lượng đa thức Chương 3. Một số dạng toán liên quan đến đa thức Để hoàn thành luận văn này, tác giả xin được gửi lời cảm ơn sâu sắc tới GS.TSKH Nguyễn Văn Mậu đã dành thời gian hướng dẫn, chỉ bảo tận tình, giúp đỡ trong suốt quá trình xây dựng đề cương cũng như hoàn thành luận văn. Tác giả xin gửi lời cảm ơn chân thành tới các quý thầy cô đã 1
  • 5. đọc, kiểm tra, đánh giá và đưa ra những ý kiến quý báu để luận văn được đầy đủ và phong phú hơn.Qua đây, tác giả xin cảm ơn Ban Giám Hiệu, phòng sau Đại học, khoa Toán Tin trường Đại Học Khoa học Tự Nhiên Hà Nội đã giảng dạy, tạo điều kiện thuận lợi trong suốt quá trình học tập. Tuy bản thân đã có nhiều cố gắng, nỗ lực nghiên cứu, song do điều kiện và trình độ còn hạn chế nên luận văn khó tránh khỏi những sai sót. Tác giả kính mong nhận được sự đóng góp ý kiến của các thầy cô để bản luận văn được hoàn thiện hơn! Tác giả xin chân thành cảm ơn! Hà Nội, tháng 10 năm 2016 Tác giả Trần Thị Viết Thủy 2
  • 6. Chương 1 Xác định đa thức 1.1 Một số tính chất cơ bản của đa thức Định nghĩa 1.1 (xem [2]). Cho vành A là một vành giao hoán có đơn vị. Ta gọi đa thức bậc n biến x là một biểu thức có dạng Pn(x) = anxn + an−1xn−1 + · · · + a1x + a0(an 6= 0), trong đó các ai ∈ A được gọi là hệ số, an là hệ số bậc cao nhất và a0 là hệ số tự do của đa thức. Nếu ai = 0, i = 0, · · · , n − 1 và a0 6= 0 thì ta có bậc của đa thức là 0. Nếu ai = 0 ∀i = 0, · · · , n − 1 thì ta coi bậc của đa thức là −∞ và gọi là đa thức không. Tập hợp tất cả các đa thức với hệ số lấy trong vành A được kí hiệu là A[x]. Khi A = K là một trường thì vành K[x] là một vành giao hoán có đơn vị. Ta thường xét A = Z,hoặc A = Q hoặc A = R. Khi đó ta có các vành đa thức tương ứng là Z[x], Q[x], R[x]. Tính chất 1.1 (xem [2]). Nếu các đa thức f(x) và g(x) nguyên tố cùng nhau và các đa thức f(x) và h(x) nguyên tố cùng nhau thì các đa thức f(x) và g(x)h(x) cũng nguyên tố cùng nhau. Tính chất 1.2 (xem [2]). Nếu các đa thức f(x), g(x), h(x) thỏa mãn điều kiện f(x)h(x) chia hết cho g(x), g(x) và h(x) nguyên tố cùng nhau thì f(x) chia hết cho g(x). 3
  • 7. Tính chất 1.3 (xem [2]). Nếu đa thức f(x) chia hết cho các đa thức g(x) và h(x) với g(x) nguyên tố cùng nhau thì f(x) chia hết cho g(x)h(x). Tính chất 1.4 (xem [2]). Nếu các đa thức f(x) và g(x) nguyên tố cùng nhau thì [f(x)]m và [g(x)]n cũng nguyên tố cùng nhau với mọi m, n nguyên dương. Định lý 1.1 (xem [7]). [Định lí về nghiệm của đa thức] Nếu một đa thức bậc n và có hệ số của số hạng có bậc cao nhất khác 0 thì nó có không quá n nghiệm. Định lý 1.2 (xem [7]). [Định lí Bezout] Cho đa thức P(x) ∈ R[x] và số thực α, khi đó α là nghiệm của P(x) khi và chỉ khi P(x) . . .(x − α). Điều này có nghĩa là tồn tại đa thức Q(x) ∈ R[x] sao cho P(x) = (x − α).Q(x). Định lý 1.3 (Công thức khai triển Abel). Cho bộ số đôi một khác nhau x1, x2, . . . , xn. Khi đó mọi đa thức P(x) với degP(x) < n+1 đều viết được dưới dạng P(x) = a0 + a1 (x − x1) + a2 (x − x1) (x − x2) + · · · + an (x − x1) (x − x2) . . . (x − xn) . Định lý 1.4 (Định lí Viet thuận). Cho đa thức P(x) = a0xn + a1xn−1 + a2xn−2 + . . . + an(a0 6= 0) có n nghiệm là x1, x2, x3, . . . , xn. Khi đó ta có                S1 = x1 + x2 + . . . + xn = − a1 a0 S2 = x1x2 + x1x3 + . . . + x1xn + . . . + xn−1xn = a2 a0 . . . . . . . . . . . . . . . . . . . . . . . . . . . . . . . . . . . . Sn = x1x2 . . . xn = (−1)n an a0 . Định lý 1.5 (Định lí Viet đảo). Ngược lại nếu có các số x1, x2, x3, . . . , xn thỏa mãn        x1 + x2 + . . . + xn = S1 x1x2 + x1x3 + . . . + x1xn + . . . + xn−1xn = S2 . . . . . . . . . . . . . . . . . . . . . . . . . . . . . . . . . . . . x1x2 . . . xn = Sn. 4
  • 8. thì x1, x2, x3, . . . , xn là các nghiệm của đa thức P(x) = xn − S1xn−1 + S2xn−2 + . . . + (−1)n Sn. Định lý 1.6 (Định lí Lagrange). Nếu f(x) là hàm liên tục trên đoạn [a, b], khả vi trên khoảng (a, b) thì tồn tại c ∈ (a, b) sao cho f0 (c) = f(b) − f(a) b − a . Một hệ quả rất quan trọng, được áp dụng nhiều trong giải toán của định lí Lagrange, đó là định lí Rolle: Định lý 1.7 (Định lí Rolle ). Cho f(x) là hàm liên tục trên đoạn [a, b], khả vi trên khoảng (a, b) và f(a) = f(b) thì tồn tại c ∈ (a, b) sao cho f0 (c) = 0. Định lý 1.8 (Bất đẳng thức Schur ). Cho các số không âm a, b, c. Khi đó với mọi r > 0 ta có bất đẳng thức ar (a − b)(a − c) + br (b − c)(b − a) + cr (c − a)(c − b) ≥ 0. Đẳng thức xảy ra khi và chỉ khi a = b = c hoặc a = b, c = 0 và các hoán vị tương ứng. Các trường hợp thường được dùng để giải toán là r = 1, r = 2. 1.2 Xác định đa thức theo các đặc trưng số học Trong phần này ta khảo sát các bài toán về xác định đa thức với hệ số nguyên và đa thức nhận giá trị nguyên trên tập số tự nhiên dựa vào các đặc trưng số học như: tính chia hết, đồng dư, nguyên tố cùng nhau, ... Bài toán 1.1 (Mathemmatical Reflection issue 4, 2015). Tìm tất cả các đa thức P(x) bậc ≥ 1 với hệ số nguyên và thỏa mãn điều kiện a2 + b2 − c2 | P(a) + P(b) − P(c), ∀a, b, c ∈ Z, (a2 + b2 − c2 6= 0). Lời giải. Ta có a2 + b2 − c2 | P(a) + P(b) − P(c), ∀a, b, c ∈ Z. (1.1) 5
  • 9. Chọn b = c, trong (1.1) ta có P(a) . . .a2 , ∀a ∈ Z. Suy ra P(a) = ma2 , ∀a ∈ Z, m ∈ Z. (1.2) Chọn b = 0, trong (1.1) ta được a2 − c2 | P(a) + P(0) − P(c), ∀a, b, c ∈ Z. (1.3) Theo định lý về phương trình Pythagoras, luôn tồn tại vô số các cặp số nguyên (a, b) sao cho a2 + b2 = m2 , m ∈ Z. Gọi tập hợp gồm các cặp số nguyên (a, b) như thế là S. Theo (1.3) ta có a2 + b2 − c2 | P( √ a2 + b2) + P(0) − P(c), ∀a, b ∈ S, c ∈ Z. (1.4) Từ (1.1) và (1.4), ta suy ra a2 + b2 − c2 | P( √ a2 + b2) + P(0) − P(a) − P(b), ∀a, b ∈ S, c ∈ Z. Hay a2 + b2 − c2 | P( √ a2 + b2) − P(a) − P(b), ∀a, b ∈ S, c ∈ Z do (1.2). Cho c → +∞ ta thu được P( √ a2 + b2) = P(a) + P(b), ∀a, b ∈ S. Vậy P( √ a2 + b2) = P(a) + P(b). Chọn a = b = x ta được P(x √ 2) = 2P(x), ∀x ∈ R. Giả sử rằng P(x) = anxn +an−1xn−1 +. . .+a1x+a0, ai ∈ Z, ∀i = 0, n, sau đó so sánh hệ số bậc cao nhất tương ứng ở hai vế ta được an( √ 2)n = 2an ⇒ n = 2. Suy ra P(x) = a2x2 + a1x + a0, do P(0) = 0 nên a0 = 0. Lại từ P(x) . . .x2 , ∀x ∈ Z nên a1 = 0. Vậy đa thức cần tìm là P(x) = kx2 , k ∈ Z tùy ý khác 0. Bài toán 1.2 (Olympic SV, 1996). Cho Pn(x) là đa thức bậc n và cho m ∈ N∗ . Chứng minh rằng 6
  • 10. Nếu Pn(xm ) chia hết cho (x − a)k thì nó chia hết cho (xm − am )k (a 6= 0). Lời giải. Giả sử, Pn(x) = an(x − am )n + · · · + a2(x − am )2 + a1(x − am ) + a0. Khi đó Pn(xm ) = an(xm − am )n + · · · + a2(xm − am )2 + a1(xm − am ) + a0. Ta chứng minh a0 = a1 = · · · = ak−1 = 0 bằng phương pháp phản chứng. Thật vậy, giả sử ai là số khác không đầu tiên, trong đó 0 6 ai 6 k−1. Dễ thấy rằng Pn(xm ) không chia hết cho (x − a)i + 1, với i + 1 > k. Suy ra Pn(xm ) không chia hết cho (x − a)k , mâu thuẫn. Suy ra điều phải chứng minh. Đặc biệt khi k = a = 1, ta có Pn(xm ) chia hết cho x − 1 thì nó chia hết cho xm − 1. Bài toán 1.3 (Olympic SV, 2002). Tồn tại hay không tồn tại một đa thức P(x) bậc 2002 sao cho P(x2 − 2001) chia hết cho P(x)? Lời giải. Ta giả sử tồn tại đa thức P(x) với deg P(x) = 2002. Xét đa thức P(x) = (x + a)2002 . Ta có P(x2 − 2001) = (x2 − 2001 + a)2002 = (x + a)2 − 2a(x + a) + a2 + a − 2001 2002 . Nếu ta chọn được a, sao cho a2 + a − 2001 = 0 hay a = −1 + √ 8005 2 hoặc a = −1 − √ 8005 2 , thì đa thức P(x2 − 2001) = (x2 − a2 )2002 = (x + a)2002 (x − a)2002 chia hết cho P(x). Vậy, đa thức P(x) = x + −1 + √ 8005 2 2002 7
  • 11. hoặc đa thức P(x) = x + −1 − √ 8005 2 2002 thoả mãn điều kiện bài toán. Lời bàn: Vì sao lại xét đa thức P(x) = (x + a)2002 như vây? Ta xét từ bài toán đơn giản trước Tồn tại hay không tồn tại một đa thức P(x) bậc 2 sao cho P(x2 − 1) chia hết cho P(x)? Xét đa thức P(x) = (x + a)2 và chỉ ra được tồn tại đa thức, do đó bài toán trên xét P(x) = (x + a)2002 . Từ đó có thể nâng bài toán với bậc của đa thức P(x) cao hơn : Tồn tại hay không tồn tại một đa thức P(x) bậc 2018 sao cho P(x2 − 2017) chia hết cho P(x)? Phát triển thành bài toán tổng quát hơn: Có tồn tại hay không tồn tại một đa thức P(x) bậc k sao cho P(x2 −k) chia hết cho P(x) ( k là số nguyên dương.) Bài toán 1.4 (HSGQG, 2015). Cho fn(x) là dãy đa thức xác định bởi f0(x) = 2, f1(x) = 3x, . . . , fn(x) = 3xfn−1(x)+(1−x−2x2 )fn−2(x) ∀n ≥ 2. Tìm tất cả các số nguyên dương n để fn(x) . . . (x3 − x2 + x). Lời giải. Từ công thức truy hồi ta có fn(x) = 3x · fn−1(x) − (x + 1)(2x − 1)fn−2(x) = (x + 1 + 2x − 1)fn−1(x) − (x + 1)(2x − 1)fn−2(x) = (x + 1)fn−1(x) + (2x − 1)[fn−1(x) − (x + 1)fn−2(x)]. Nên fn(x) − (x + 1)fn−1(x) = (2x − 1)[fn−1(x) − (x + 1)fn−2(x)] fn−1(x) − (x + 1)fn−2(x) = (2x − 1)[fn−2(x) − (x + 1)fn−3(x)] fn−2(x) − (x + 1)fn−3(x) = (2x − 1)[fn−3(x) − (x + 1)fn−4(x)] . . . . . . . . . . . . . . . . . . . . . . . . . . . . . . . . . . . . f2(x) − (x + 1)f1(x) = (2x − 1)[f1(x) − (x + 1)f0(x)]. 8
  • 12. Suy ra fn(x) − (x + 1)fn−1(x) = (2x − 1)n−1 [f1(x) − (x + 1)f0(x)] = (2x − 1)n−1 (x − 2). Do đó fn(x) − (2x − 1)n = (x + 1)fn−1(x) + (2x − 1)n−1 (x − 2) − (2x − 1)n . Vì vậy, fn(x) − (2x − 1)n = (x + 1)fn−1(x) + (2x − 1)n−1 (x − 2 − 2x + 1) = (x + 1)[fn−1(x) − (2x − 1)n−1 ] = (x + 1)n [f0(x) − (2x − 1)0 ] = (x + 1)n . Vậy fn(x) = (2x−1)n +(x+1)n . Đặt Q(x) = x3 −x2 +x = x(x2 −x+1). Vì fn(x) . . . Q(x) nên fn(0) = 0 hay 1n + (−1)n = 0 nên n lẻ. fn(−2) = (−5)n + (−1)n = −(5n + 1) (do n lẻ) chia hết cho Q(−2) = −2[(−2)2 − (−2) + 1] = (−2)7. Vì fn(−2) = −(5n + 1) lại là số chẵn nên fn(−2) . . . 7. Do 125 ≡ −1 (mod 7) nên xét các trường hợp sau • n = 3k, k lẻ ta có 5n + 1 = 53k + 1 ≡ (−1)k + 1 ≡ 0 (mod 7), • n = 3k + 1, k chẵn ta có 5n + 1 = 5 · 53k + 1 ≡ 6 (mod 7), • n = 3k + 2, k lẻ ta có 5n + 1 = 25 · 53k + 1 ≡ −24 ≡ 3 (mod 7). Suy ra điều kiện cần của n là n = 3k với k lẻ. Khi đó fn(x) = (x + 1)3k + (2x − 1)3k chia hết cho (x + 1)3 + (2x − 1)3 , ∀k. Nhận thấy (x + 1)3 + (2x − 1)3 = (9x3 − 9x2 + 9x) . . . g(x) nên ta có x = 3k với k là số tự nhiên lẻ thỏa mãn, đặt k = 2m + 1 với m nguyên dương thì n = 6n + 3. Vậy tất cả các số n cần tìm dạng 6m + 3 với m là số nguyên dương. Nhận xét: Ta có thể tìm ra được công thức tổng quát của dãy đa thức đã cho bằng cách coi x là hằng số và xây dựng số hạng của dãy số 9
  • 13. tương ứng, tức là xét dãy số u0 = 2, u1 = 3x, un = 3xun−1 + (1 − x − x2 )un−2 với x là tham số thực nào đó. Xét phương trình đặc trưng t2 − 3xt + 2x2 + x − 1 = 0, suy ra t = x + 1, t = 2x − 1. Do đó un = α(x + 1)n + β(2x − 1)n .Dựa vào u0, u1 ta tìm được α = β = 1. Do vậy, fn(x) = (2x − 1)n + (x + 1)n . Bài toán thuộc dạng về tính chia hết của đa thức kết hợp với đa thức xác định bởi hệ thức truy hồi. Dưới đây là một bài tương tự: Cho fn(x) là dãy đa thức xác định bởi công thức f0(x) = 2, f1(x) = 2x + 2, fn+2(x) = (2x + 2)fn+1(x) − (x2 + 2x − 3)fn(x), n ≥ 1 Tìm tất cả các giá trị của n sao cho fn(x) chia hết cho x2 + 2x + 5. Bài toán 1.5 (IMO Shortlisted 2002). Cho m, n ∈ N(m, n 2) và các số nguyên a1, a2, ..., an sao cho không có số nào trong chúng chia hết cho mn−1 . Chứng minh rằng tồn tại các số nguyên e1, e2, ..., en không đồng thời bằng 0 sao cho |ei| m, ∀i và e1a1 + e2a2 + · · · + enan chia hết cho mn . Lời giải. Giả sử không tồn tại các số nguyên e1, e2, ..., en không đồng thời bằng 0 sao cho |ei| m, ∀i và e1a1 + e2a2 + · · · + enan chia hết cho mn . Xét tập A gồm các số có dạng n P i=1 eiai, với 0 ≤ ei ≤ m − 1, i = 1, n. Trong A có tất cả mn số và do giả thiết phản chứng ta suy ra mn số này lập thành hệ thặng dư đầy đủ (mod mn ) (vì nếu không như vậy thì có hai số cùng số dư khi chia cho mn và hiệu hai số này thỏa mãn đề bài. Vô lý). Xét f(x) = P a∈A xa , ta thấy f(x) = n Y i=1 m−1 X j=0 xjai = n Y i=1 1 − xmai 1 − xai . (1.5) 10
  • 14. Xét số phức ε = cos 2π mn + i sin 2π mn . Do mn phần tử của A lập thành hệ thặng dư đầy đủ (mod mn ) nên ta phải có f(ε) = 0. Do đó từ (1.5)ta suy ra n Y i=1 (1 − εmai ) = 0. Tuy nhiên điều này mâu thuẫn với giả thiết các số a1, a2, ..., an là các số nguyên sao cho không có số nào trong chúng chia hết cho mn−1 . Điều đó chứng tỏ phản chứng là sai. Vậy bài toán được chứng minh. Bài toán 1.6 (Olympic SV, 2004). Xác định đa thức f(x) = x5 − 3x4 + 2x3 + ax2 + bx + c biết rằng nó chia hết cho đa thức (x − 1)(x + 1)(x − 2). Lời giải. Ta có f(x) chia hết cho (x − 1)(x + 1)(x − 2) khi và chỉ khi f(1) = a + b + c = 0, f(−1) = a − b + c − 6 = 0, f(2) = 4a + 2b + c = 0. Giải hệ này ta thu được a = 1, b = −3, c = 2. Vậy đa thức cần tìm là f(x) = x5 − 3x4 + 2x3 + x2 − 3x + 2. Bài toán 1.7. Tìm tất cả các đa thức P(x) bậc 5 thỏa mãn các điều kiện sau: Đa thức (P(x) + 1) chia hết cho (x − 1)3 và đa thức (P(x) − 1) chia hết cho (x + 1)3 . Lời giải. Từ giả thiết suy ra deg P0 (x) = 4 và P0 (x) chia hết cho (x−1)2 và (x + 1)2 . Vậy nên P0 (x) = a(x − 1)2 (x + 1)2 = a(x4 − 2x2 + 1) và P(x) = a x5 5 − 2 x3 3 + x + b . 11
  • 15. Kết hợp với điều kiện P(1) = −1 và P(−1) = 1 ta thu được P(x) = − 1 8 (3x5 − 10x3 + 15). Thử lại ta thấy nghiệm này không thỏa mãn. Vậy không tồn tại đa thưc bậc 5 thỏa mãn điều kiện bài ra. Bài toán 1.8. Tìm đa thức bậc 3 dạng f(x) = x3 + ax2 + bx + c sao cho f(x) chia hết cho (x − 2) và f(x) chia cho x2 − 1 thì dư 2x. Lời giải. Vì f(x) chia hết cho x − 2 nên f(2) = 8 + 4a + 2b + c = 0. Do f(x) chia cho x2 − 1 thì dư 2x nên g(x) = f(x) − 2x chia hết cho x2 − 1. Suy ra g(1) = 1 + a + (b − 2) + c = 0 hay a + b + c = 1 và g(−1) = −1 + a − b + 2 + c = 0 hay a − b + c = −1. Từ đó ta nhận được a = −10, c = −10, b = −19. Vậy đa thức cần tìm có dạng f(x) = x3 − 10x2 − 19x − 10. Bài toán 1.9. Xác định đa thức bậc n dạng f(x) = xn + an−1xn−1 + · · · + a1x + a0 biết rằng khi chia f(x) cho (x − b1), (x − b2), · · · , (x − bn)(bi ∈ Z, bi 6= bj nếu i 6= j)đều có chung số dư là m(m ∈ Z). 12
  • 16. Lời giải. Từ giả thiết suy ra f(bi) = m(i = 1, 2, · · · , n). Đặt f(x) − m = g(x) thì deg g = n và hệ số cao nhất của g(x) bằng 1 và g(x) có n nghiệm phân biệt là b1, b2, · · · , bn. Xét đa thức h(x) = g(x) − (x − b1)(x − b2) · · · (x − bn). Khi đó h(x) = (an−1 − A1)xn−1 + (an−2 − A2)xn−2 + · · · + (a0 − An−m), trong đó A1, A2, · · · , An được xác định như sau        A1 = (−1)1 (b1 + b2 + · · · + bn) A2 = (−1)2 (b1b2 + b1b3 + · · · + bn−1bn) . . . . . . . . . . . . . . . . . . . . . . . . . . . . . . . . . . . . An = b1 + b2 + · · · + bn. 1.3 Xác định đa thức theo các đặc trưng nghiệm Bài toán 1.10 (Kì thi chọn đội tuyển HSG TPHCM, 2012 - 2013). Tìm tất cả các đa thức P(x) hệ số thực thỏa mãn điều kiện P(x) · P(x − 3) = P(x2 ), ∀x ∈ R. (1.6) Lời giải. TH1: P(x) ≡ C (C là hằng số thực) thỏa mãn (1.6). Suy ra c2 = c nên c = 0 hoặc c = 1. Do đó P(x) = 0 hoặc P(x) = 1. TH2: deg P(x) ≥ 1. Gọi α là một nghiệm phức tùy ý của P(x). Từ (1.6) thay x = α ta có P(α2 ) = 0, suy ra x = α2 cũng là một nghiệm của P(x). Từ đó có α, α2 , α4 , . . . , α2n cũng các nghiệm của P(x) mà P(x) chỉ có hữu hạn nghiệm (do đang xét P(x) khác đa thức không), suy ra h |α| = 0 |α| = 1. (I) Từ (1.6) lại thay x = α + 3, ta có P((α + 3)2 ) = 0, suy ra x = (α + 3)2 là nghiệm của P(x). Từ x = (α + 3)2 là nghiệm của P(x), tương tự phần 13
  • 17. trên có (α + 3)2 , (α + 3)4 , (α + 3)8 , (α + 3)16 , . . . là các nghiệm của P(x) mà P(x) chỉ có hữu hạn nghiệm, suy ra h |(α + 3)|2 = 0 |α + 3|2 = 1. Hay h |(α + 3)| = 0 |α + 3| = 1. (II) Như vậy, nếu α là nghiệm của P(x) thì ta có α thỏa mãn hệ (I) và (II). Từ biểu diễn số phức α thỏa mãn (I) và (II) trên mặt phẳng phức ta thấy hệ trên không có nghiệm. Suy ra không tồn tại đa thức hệ số thực P(x) bậc lớn hơn hoặc bằng 1 thỏa mãn (1.6). Kết luận. Các đa thức P(x) hệ số thực thỏa mãn (1.6) là P(x) = 0 hoặc P(x) = 1. Bài toán 1.11 (Moldova MO 2004). Tìm đa thức P(x) hệ số thực thỏa mãn điều kiện (x3 +3x2 +3x+2)P(x−1) = (x3 −3x2 +3x−2)P(x), ∀x ∈ R. Lời giải. Trước hết ta tìm nghiệm của P(x). Từ giả thiết, ta có (x + 2)(x2 + x + 1)P(x − 1) = (x − 2)(x2 − x + 1)P(x). (1.7) Từ đây, ta chọn x = −2 sẽ được P(−2) = 0. Chọn x = −1 được P(−1) = 0, chọn x = 0 được P(0) = 0 và khi x = 1 thì được P(1) = 0. Suy ra P(x) = x(x − 1)(x + 1)(x + 2)Q(x), với Q(x) là đa thức hệ số thực. Thay P(x) vào (1.7), ta được (x2 + x + 1)Q(x − 1) = (x2 − x + 1)Q(x), ∀x 6= {0, ±1, ±2} ⇔ Q(x − 1) x2 − x + 1 = Q(x) (x2 + x + 1) , ∀x 6= {0, ±1, ±2} ⇔ Q(x − 1) (x − 1)2 + (x − 1) + 1 = Q(x) (x2 + x + 1) , ∀x 6= {0, ±1, ±2}. Đặt R(x) = Q(x) (x2 + x + 1) , ta có R(x) = R(x−1) với mọi ∀x 6= {0, ±1, ±2}. Suy ra R(x) ≡ C hằng số nên Q(x) = C(x2 + x + 1). Do đó P(x) = Cx(x − 1)(x + 1)(x + 2). Thử lại ta thấy thỏa mãn. Vậy P(x) = Cx(x − 1)(x + 1)(x + 2). 14
  • 18. Bài toán 1.12 (Bulgary MO, 2004). Tìm tất cả các cặp đa thức P(x), Q(x) thuộcR[x] bậc ≥ 1 và thỏa mãn điều kiện P(x)Q(x + 1) ≡ P(x + 2017)Q(x). Lời giải. Đặt R(x) = P(x)P(x + 1) · · · P(x + 2016), ta có P(x) · Q(x + 1) ≡ P(x + 2017)Q(x) Suy ra Q(x) Q(x + 1) = P(x) P(x + 2017) = P(x)P(x + 1) · · · P(x + 2016)) P(x + 1)P(x + 2) · · · P(x + 2017) = R(x) R(x + 1) . Nếu x lớn hơn nghiệm lớn nhất của P(x) thì Q(x) R(x) = Q(x + 1) R(x + 1) . Suy ra, với mọi số tự nhiên n, ta có Q(x) R(x) = Q(x + n) R(x + n) . Cho n → +∞ thì Q(x + n) R(x + n) → c 6= 0. Do đó Q(x) R(x) = c hay Q(x) = c.R(x). Vậy Q(x) = c.P(x).P(x + 1) · · · P(x + 2016). Bài toán 1.13 (Poland MO). Cho đa thức P(x) có bậc n 1 có n nghiệm thực x1, x2, . . . , xn phân biệt. Chứng minh rằng 1 P0(x1) + 1 P0(x2) + . . . + 1 P0(xn) = 0. Lời giải. Đặt P(x) = a(x − x1)(x − x2) . . . (x − xn), a 6= 0, suy ra P0 (x) = P1(x) + P2(x) + . . . + Pn(x) với Pi(x) = n Y j=1 (x − xj). 15
  • 19. Ta thấy Pi(xj) = 0, ∀i 6= j. Suy ra P0 (xj) = Pj(xj) 6= 0, ∀j = 1, n. Xét đa thức F(xi) = n P i=1 Pi(xi) P0(xi) − 1 = 0 có bậc không quá n − 1. Với i = 1, n, ta có F(xi) = Pi(xi) P0(xi) − 1 = 0, suy ra F(x) có n nghiệm phân biệt. Vậy nên F(x) ≡ 0. Lại có hệ số của F(x) ứng với xn−1 bằng 0 nên a P0(x1) + a P0(x2) + . . . + a P0(xn) = 0. Suy ra 1 P0(x1) + 1 P0(x2) + . . . + 1 P0(xn) = 0. Tiếp theo xét bài toán sử dụng khá nhiều kĩ thuật: sử dụng định lí Vi-et nhưng khéo léo trong việc đổi dấu các nghiệm nên không còn xuất hiện (−1)n , sử dụng đánh giá bất đẳng thức ∀x, y ≥ 1 ta luôn có xy + 1 x + 1 y ≥ 1 xy + x + y. và sử dụng phương pháp quy nạp toán học. Bài toán 1.14 (Moscow MO, 2011). Cho n ∈ N, n ≥ 3. Tìm tất cả các đa thức hệ số thực f(x) = a0 + a1x + · · · + anxn , (an 6= 0) có n nghiệm không lớn hơn −1 và thỏa mãn điều kiện a2 0 + a1an = a2 n + a0an−1. Lời giải. Vì f(x) có n nghiệm không lớn hơn −1 nên f(x) = a0 + a1x + . . . + anxn = an(x + x1)(x + x2) . . . (x + xn). Với mọi xi ≥ 1, ∀i = 1, n. Đặt                S1 = x1 + x2 + x3 + . . . + xn = an−1 an S2 = x1x2 + x2x3 + . . . + xn−1xn = an−2 an . . . . . . . . . . . . . . . . . . . . . . . . . . . . . . . . . . . . Sn = x1x2x3 . . . xn = a0 an . 16
  • 20. Ta có a2 0 + a1an = a2 n + a0an−1 tương đương với a0 an 2 + a1 an = 1 + a0 an . an−1 an . S2 n + Sn−1 = 1 + SnS1 ⇔ Sn + Sn−1 Sn = 1 Sn + S1. Do đó x1x2 . . . xn + 1 x1 + . . . + 1 xn = 1 x1x2 . . . xn + x1 + . . . + xn. mà x1x2 . . . xn+ 1 x1 +. . .+ 1 xn ≥ 1 x1x2 . . . xn +x1+. . .+xn.( chứng minh theo qui nạp) Dấu bằng xảy ra khi và chỉ khi có n − 1 số bằng 1. Vậy đa thức cần tìm là f(x) = an(x + 1)n−1 (x + a) với a là hằng số lớn hơn hoặc bằng 1. Bài toán 1.15 (Olympic SV, 2000). Cho a, b ∈ R. Tìm tất cả các đa thức P(x) thoả mãn điều kiện xP(x − a) = (x − b)P(x), ∀x ∈ R. Lời giải. i) Khi a = 0, b = 0 thì P(x) tuỳ ý. ii) Khi a = 0, b 6= 0 thì P(x) = 0 ∀x. iii) Khi a 6= 0, b = 0 thì P(x) =const tuỳ ý. iv) Khi a 6= 0, b 6= 0 thì: a) Nếu b a 6∈ N, thì khi thay x = b vào ta được x = b − a là nghiệm. Tương tự khi thay x = b − a thì sẽ có x = b − 2a là nghiệm,... Suy ra P(x) = x, ∀x ∈ R. b) Nếu b a ∈ N thì P(x) có x = a, x = 2a, . . . , x = (n−1)a là nghiệm. Suy ra P(x) = (x − a)(x − 2a) . . . (x − (n − 1)a)Q(x). 17
  • 21. Thế vào điều kiện bài ra, ta được Q(x − a) = Q(x), ∀x ∈ R, hay Q(x) = const . Vậy nên P(x) = (x − a)(x − 2a) . . . [x − (n − 1)a]. Bài toán 1.16 (Olympic 30-4, THPT chuyên Tiền Giang đề nghị). Gọi xi, i = 1, 2011 là các nghiệm của đa thức P(x) = x2011 + 2011x2000 + a2009x2009 + · · · + a0. Biết rằng x64 1 + x64 2 + · · · + x64 2011 = 2011. Hãy xác định đa thức P(x). Lời giải. Áp dụng định lí Viet, ta có n−1 P i=1 xi = −2011. Áp dụng bất đẳng thức Cauchy, ta có 20112 = 2011 X i=1 xi 2 ≤ 2011 2011 X i=1 x2 i , suy ra 20114 ≤ 20112 2011 P i=1 x2 i 2 . Lại áp dụng bất đẳng thức Cauchy, ta có 2011 X i=1 x2 i 2 ≤ 2011 2011 X i=1 x4 i , suy ra 20114 = 2011 P i=1 xi 4 ≤ 20113 2011 P i=1 x4 i . Biến đổi tương tự, ta thu được 2011 X i=1 xi 8 ≤ 20117 2011 X i=1 x8 i . Vậy nên 2011 X i=1 xi 64 ≤ 201163 2011 X i=1 x64 i = 201164 . (1.8) Mặt khác, n−1 P i=1 xi = −2011, nên từ (1.8) suy ra bất đẳng thức Cauchy xảy ra dấu bằng. Chứng tỏ các nghiệm bằng nhau và bằng -1. Do đó đa thức cần tìm là P(x) = (x + 1)2011 . 18
  • 22. 1.4 Xác dịnh đa thức theo phép biến đổi vi phân hàm Trong phần này ta khảo sát một số dạng toán về xác định đa thức theo phép biến đổi vi phân hàm. Bài toán 1.17 (Olympic SV, 1993). Cho p(x) (6= const) là đa thức với hệ số thực. Chứng minh rằng nếu hệ phương trình        x R 0 p(t) sin tdt = 0, x R 0 p(t) cos tdt = 0. có nghiệm thực thì số nghiệm thực chỉ có thể là hữu hạn. Lời giải. Gọi p(k) (t) là đạo hàm cấp k của p(t) (p0 (t) = p(t)) và ký hiệu Uk = x Z 0 p(k) (t) sin tdt, Vk = x Z 0 p(k) (t) cos tdt. Giả sử deg p = n. Suy ra Uk = 0, Vt = 0 nếu K n. Sử dụng công thức tích phân từng phần, ta thu được        Uk = −p(k) (t) cos t x 0 + x R 0 p(k+1) (t) sin tdt Vk = p(k) (t) sin t x 0 − x R 0 p(k+1) (t) sin tdt. Suy ra Uk = −p(k) (t) cos t x 0 + Vk+1 Vk = p(k) (t) sin t x 0 − Uk+1 Ta có tiếp Uk = −p(k) (t) cos t x 0 + p(k+1) (t) sin t x 0 − Uk+2 Vk = p(k) (t) sin t x 0 + p(k+1) (t) cos t x 0 − Vk+2 , ∀k ∈ N. 19
  • 23.          U0 = − 2k6n P k=0 p(2k) (t) cos t x 0 + (2k+1)6n P k=0 p(2k+1) (t) sin t x 0 V0 = 2k6n P k=0 p(2k) (t) sin t x 0 + (2k+1)6n P k=0 p(2k+1) (t) cos t x 0 . đặt p1(t) = 2k+16n X k=0 p(2k) (t) Suy ra deg p1 = n. p2(t) = 2k+16n X k=0 p(2k+1) (t). Suy ra deg p2 = n − 1. Khi đó, (1) được viết dưới dạng U0 = −p1(t) cos t x 0 + p2(t) sin t x 0 V0 = p1(t) sin t x 0 + p2(t) cos x 0 Gọi X là tập nghiệm của hệ đã cho, tức hệ U0 = 0 V0 = 0. Với mọi x ∈ X ta có −p1(t) cos t|x 0 + p2 sin t|x 0 = 0 p1(t) sin t|x 0 + p2 cos t|x 0 = 0. Đặt P1(0) = a, P2(0) = b. Khi đó p2(x) sin x − p1(x) cos x = −a p2(x) cos x + p1(x) sin x = b. Suy ra (p2(x) sin x − p1(x) cos x)2 + (p2(x) cos x + p1(x) sin x)2 = a2 + b2 . Do đó p2 1(x) + p2 2(x) − (a2 + b2 ) = 0. Gọi Y là tập nghiệm của đa thức Q(x) = p2 1(x) + p2 2(x) − (a2 + b2 ). 20
  • 24. Suy ra X ⊂ Y . Từ deg Q = 2n suy ra |X| 6 |Y | 6 2n. Tức X chỉ có hữu hạn phần tử. Cách khác. Ta có thể sử dụng số phức để giải bài toán. Viết lại hệ dưới dạng F(x) := x Z 0 P(t)eit dt = 0. Ta có F0 (x) = P(x)eix nên phương trình F0 (x) = 0 chỉ có hữu hạn nghiệm. Suy ra phương trình F(x) = 0 có không quá hữu hạn nghiệm. Bài toán 1.18 (Olympic SV,1994). a) Cho hàm số f : [a, b] → [a, b], với a b và thỏa mãn điều kiện |f(x) − f(y)| |x − y|, ∀x, y ∈ [a, b] và x 6= y. Chứng minh rằng phương trình f(x) = x có duy nhất một nghiệm thuộc [a, b]. b) Cho hàm số f(x) khả vi trên [a, b], có không điểm trên [a, b] và thỏa mãn |f0 (x)| |f(x)|, ∀x ∈ [a, b]. Chứng minh rằng f(x) = 0, ∀x ∈ [a, b]. Lời giải. a) Xét hàm số g(x) = f(x) − x. Ta thấy g(x) liên tục trên [a, b]. Do đó tồn tại x0 ∈ [a, b] sao cho g(x0) = min x∈[a,b] g(x). (1.9) Ta sẽ chứng minh rằng g(x0) = 0. Thật vậy, giả sử g(x0) 6= 0 và vì vậy, f(x0) 6= x0. Từ bất đẳng thức đã cho, ta có f(f(x0)) − f(x0) |f(x0) − x0|. Suy ra f(x0) g(x0) 21
  • 25. Điều này mâu thuẫn với (1.9), nghĩa là f(x0) = x0. Giả sử phương trình f(x) = x còn có nghiệm x1 với x0 6= x1 ∈ [a, b]. Ta có x1 6= x0 x1 ∈ [a, b]. Suy ra |f(x1) − f(x0)| = |x1 − x0|, Mâu thuẫn với bất đẳng thức đã cho. Tóm lại, phương trình f(x) = x có duy nhất nghiệm trên [a, b]. b) Giả sử x0 là nghiệm của phương trình f(x) = 0 với x0 ∈ [a, b]. Theo khai triển Taylor tại x0, thì f(x) = f(x0) + f0 (c)(x − x0) = f0 (c)(x − x0). Xét khoảng đóng G := h x0 − 1 2 , x0 + 1 2 i ∩ [a, b]. Vì f(x) khả vi trên [a, b] nên f(x) đạt cực đại trên đoạn đóng G. Giả sử |f(xm)| = max x∈G |f(x)|, xm ∈ G. Suy ra |f(xm)| = |f0 (cm)| |xm − x0| 6 |f(cm)| |xm − x0| 6 1 2 |f(cm)| 6 1 2 |f(xm)|. Hay f(x) = 0 với mọi x ∈ G. Như vậy, nếu tại một điểm trên [a, b] mà f(x) = 0 thì f(x) = 0 trên toàn bộ lân cận với bán kính bằng 1/2 của điểm đó. Bằng việc xét các điểm x0 khác nhau (mà tại đó f(x0) = 0) lan dần về hai phía của đoạn [a, b] thì sau một số hữu hạn bước ta sẽ được f(x) = 0 với ∀x ∈ [a, b]. Bài toán 1.19 (Olympic SV, 1995). Xét đa thức Pn(x) = 1 2nn! dn dxn [x2 − 1)n ] Chứng minh rằng nếu f(x) là đa thức bậc m (m n) thì 1 Z −1 f(x)Pn(x)dx = 0. 22
  • 26. Lời giải. Sử dụng công thức tích phân từng phần 1 Z −1 d(n) (x2 − 1)n dxn f(x)dx = = d(n−1) (x2 − 1)n dxn−1 f(x)|1 −1 − 1 Z −1 d(n−1) (x2 − 1)n dxn−1 f0 (x)dx = · · · = 1 Z −1 (x2 − 1)n f(n) (x)dx = 1 Z −1 0dx = 0. Bài toán 1.20 (Olympic SV, 1996). Cho g(x) là một đa thức bậc 1996. Biết rằng, ứng với mọi x ∈ R, ta đều có g(x + h) = g(x) + hg0 (x + h)θ(x, h), trong đó θ(x, h) bị chặn và g00 (x) 6= 0. Tính lim h→0 θ(x, h). Lời giải. Với x xác định, ta khai triển Taylor với đa thức f(x) = g(x + h) tại h = 0 : g(x + h) = g(x) + g0 (x)h + g00 (x) 2 h2 + g 000 (x) 3! h3 + · · · + g1996 (x)h1996 1996! (do f0 (0) = g0 (x), . . . , f(1996) (0) = g(1996) (x)). Theo đề bài thì g(x + h) = g(x) + hg0 (x + hθ(x, h)). Do vậy hg0 (t + hθ(x, h)) = hg0 (x) + h2 2 g00 (x) + · · · + h1996 1996! g(1996) (x). Khai triển Taylor bậc 2 với hàm g0 (x + hθ(x, h)) tại điểm h = 0. g0 (x + hθ(x, h)) = g0 (x) + g00 (x)hθ(x, h) + 0(hθ(x, h)), nên hg0 (x + hθ(x, h)) = hg0 (x) + h2 g00 (x)θ(x, h) + h0(hθ(x, h)) = = hg0 (x) + h2 2 g00 (x) + · · · + h1996 1996! g(1996) (x). 23
  • 27. Suy ra g00 (x)θ(x, h) + 0(hθ(x, h)) h = = 12 2 g00 (x)+ h 3! g 000 (x)+· · ·+ h1996 1996! g(1996) (x) = lim h→0 g00 (x)θ(x, h) = 1 2 g00 (x). Do g00 (x) 6= 0 và lim n→0 θ(x, h) = 1 2 , nên lim n→0 θ(x, h) = 1 2 . Bài toán 1.21 (Olympic SV, 1997). Chứng minh rằng, với mọi t 0, phương trình x3 + tx − 8 = 0 luôn có nghiệm dương duy nhất, ký hiệu là x(t). Tính tích phân t Z 0 [x(t)]2 dt. Lời giải. Xét f(x) = x3 + tx − 8. Ta có f0 (x) 0, ∀x 0. Mặt khác, ta có f(0) = −8 0 và lim x→∞ f(x) = +∞. Vậy nên phương trình f(x) = 0 có nghiệm dương duy nhất. Từ phương trình x3 + tx − 8 = 0, ta có t = x2 − 8 x . Khi t = 0 thì x = 2. Ta thu được t = 7 và x3 + tx − 8 = (x − 1)(x2 + x + t) = 0. Suy ra x = 1 và 7 Z 0 [x(t)]2 dt = − 1 Z 2 x2 d x2 − 8 x = 2 Z 1 x2 2x + 8 x2 dx = = 2 Z 1 (2x3 + 8)dx = x4 2 2 1 + 8x 2 1 = − 31 2 . 24
  • 28. Bài toán 1.22 (Olympic SV, 1998). Xét các đa thức P(x) với hệ số thực thỏa mãn các điều kiện P(0) = P(1) = 0, 1 Z 0 |P0 (x)|dx = 1. Chứng minh rằng |P(x)| 6 1 2 , ∀x ∈ [0, 1]. Lời giải. Ta sử dụng phương pháp chứng minh phản chứng. Giả sử ∃x0 ∈ [0, 1] sao cho |P(x0)| 1 2 . Do P(x) liên tục tại x0, nên suy ra 1 Z 0 |P0 (x)|dx = x0 Z 0 |P0 (x)|dx + 1 Z x0 |P0 (x)|dx x0 Z 0 P0 (x)dx + 1 Z x0 P0 (x)dx =|P(x0) − P(0)| + |P(1) − P(x0)| 2|P(x0)| 1, mâu thuẫn với giả thiết 1 Z 0 |P0 (x)|dx = 1. Vậy điều giả sử là sai. Vậy nên |P(x)| 6 1 2 , ∀x ∈ [0, 1]. Bài toán 1.23 (Olympic SV, 1999). Giả sử đa thức với hệ số thực P(x) = a0 + a1x + · · · + anxn có n nghiệm thực phân biệt. Chứng minh rằng ak−1ak+1 a2 k, ∀k ∈ {1, 2, . . . , n − 1}. Lời giải. 25
  • 29. Ta sẽ chứng minh rằng [Q0 (x)]2 − Q(x)Q00 (x) 0, ∀x ∈ R, (1.10) ứng với Q(x) ∈ R(x), deg Q(x) = m và Q(x) có m nghiệm thực đơn. Ta có Q(x) = a m Y i=1 (x − αi), αi 6= αj (i 6= j). Suy ra Q0 (x) Q(x) = m X i=1 1 x − αi và [Q0 (x)]2 − Q(x)Q00 (x) Q2(x) = m X i=1 1 (x − αi)2 . (1.11) a) Nếu với t ∈ R mà Q(t) = 0 thì [Q0 (t)]2 − Q(t)Q00 (t) = [Q0 (t)]2 0 (do Q0 (t) 6= 0 và do t là nghiệm đơn). b) Nếu với t ∈ R mà Q(t) 6= 0 thì từ (1.11) suy ra (1.10). Bây giờ ta xét đa thức Q(x) = P(k) (x), k = 0, 1, . . . , n − 1. Các đa thức đó đều có nghiệm thực đơn (định lý Role). Suy ra P(k−1) (0)P(k+1) (0) [P(k) ]2 hay (k − 1)!ak−1(k + 1)!ak+1 (akk!)2 nên ak−1ak+1 (ak)2 k k + 1 a2 k. Bài toán 1.24 (Olympic SV, 2001). Cho hàm số f(x) khả vi trên đoạn [a, b] và thoả mãn điều kiện [f(x)]2 + [f0 (x)]2 0, ∀x ∈ [a, b]. 26
  • 30. Chứng minh rằng số các nghiệm của phương trình f(x) = 0 trên đoạn [a, b] là hữu hạn. Lời giải. Giả sử ngược lại, phương trình f(x) = 0 có vô số nghiệm {xn} ∈ [a, b], n = 1, 2, 3, . . . Khi đó, tồn tại dãy con {xnk } → α ∈ [a, b]. Do f(x) liên tục nên f(α) = 0. Từ giả thiết [f(x)]2 + [f0 (x)]2 0, ∀x ∈ [a, b], suy ra f0 (α) 6= 0. Mặt khác, f0 (α) = lim x→α f(x) − f(α) x − α 6= 0. Điều này chứng tỏ f(x) 6= 0 trong một lân cận nào đó của điểm α, mâu thuẫn với giả thiết α là điểm tụ của dãy {xn}n=1,2,3,.... 27
  • 31. Chương 2 Ước lượng đa thức 2.1 Đa thức Chebyshev và các tính chất Panuty Chebyshev là tên của một nhà toán học người Nga. Các kết qủa về dãy đa thức trực giao (orthogonal polynomials) có liên hệ sâu sắc đến công thức de Moivre (de Moivre’s formula). Các dãy đa thức trực giao này hoàn toàn có thể xác định được bằng công thức truy hồi giống như dãy số Fibonacci và dãy số Lucas. Đa thức này gồm có hai loại là: Ta ký hiệu đa thức Chebyshev loại I là Tn. Chữ T được chọn làm ký hiệu vì tên của Chebyshev trong tiếng Pháp là Tchebycheff và trong tiếng Đức là Tschebyscheff. Ta ký hiệu đa thức Chebyshev loại II là Un. Định nghĩa 2.1 (Đa thức Chebyshev loại I). Các đa thức Tn (x) được xác định bởi: T0 (x) = 1; T1 (x) = x Tn+1 (x) = 2x.Tn (x) − Tn−1 (x) , n ≥ 1 được gọi là đa thức Chebyshev loại I. Định nghĩa 2.2 (Đa thức Chebyshev loại II). Các đa thức Un (x) được xác định bởi: U0 (x) = 1; U1 (x) = 2x Un+1 (x) = 2x.Un (x) − Un−1 (x) , n ≥ 1 được gọi là đa thức Chebyshev loại II. Bằng phương pháp quy nạp toán học, ta dễ dàng chứng minh được: Tn (cos α) = cos nα, ∀α ∈ R; Un (cos α) = sin (n + 1) α sin α , ∀α 6= kπ, k ∈ Z. Tính chất 2.1. Tn (x) = cos (n arccos x) , ∀x ∈ [−1; 1] ; 28
  • 32. Un (x) = sin (n arccos x) √ 1 − x2 , ∀x ∈ (−1; 1) . Tính chất 2.2. Tn (x) , Un (x) ∈ Z [x] có bậc là n và hệ số cao nhất tương ứng là 2n−1 và 2n . Tính chất 2.3. Tn (x) , U (x) là các hàm số chẵn khi n chẵn và là các hàm số lẻ khi n lẻ. Tính chất 2.4. Tn (x) , Un (x) có đúng n nghiệm thực phân biệt tương ứng là: cos (2k + 1) π 2n , k = 0; n − 1 và cos kπ n + 1 , k = 1, n. Chứng minh. Do x ∈ [−1; 1] nên ta đặt x = cos α với α ∈ [0; π]. Tn (x) = 0 tương đương Tn (cos α) = 0 suy ra cos nα = 0 do đó nα = π 2 + kπ hay α = π 2n + k n π. Do α ∈ [0; π] nên 0 ≤ π 2n + k n π ≤ π ⇔ 0 ≤ 1 2n + k n ≤ 1 ⇔ − 1 2 ≤ k ≤ 2n − 1 2 = n − 1 2 . Mà k ∈ Z nên k = 0, n − 1. Vậy Tn (x) có đúng n nghiệm thực phân biệt tương ứng là: cos (2k + 1) π 2n , k = 0; n − 1. Ta chứng minh tương tự cho trường hợp của Un (x). Tính chất 2.5. |Tn (x)| ≤ 1, ∀x ∈ [−1; 1] và |Tn (x)| = 1 có đúng n + 1 điểm xk = cos kπ n , k = 0; n. Chứng minh. Theo cách đặt trên thì |Tn (x)| = |Tn (cos α)| = |cos nα| ≤ 1, ∀x ∈ [−1; 1]. |Tn (x)| = 1 ⇔ |cos nα| = 1 ⇔ sin nα = 0 ⇔ nα = kπ ⇔ α = kπ n . Do α ∈ [0; π] nên 0 ≤ kπ n ≤ π ⇔ 0 ≤ k ≤ n. Mà k ∈ Z nên k = 0; n. Lưu ý. Các điểm cos kπ n , k = 0; n nói ở trên là các điểm luân phiên Chebyshev (gọi tắt là các luân điểm). Nhận xét 2.1. Tn cos kπ n = (−1)k . 29
  • 33. Tính chất 2.6. Tn 1 2 x + 1 x = 1 2 xn + 1 xn , ∀n ∈ N, ∀x 6= 0; Un 1 2 x + 1 x = xn+1 − 1 xn+1 x − 1 x , ∀n ∈ N, ∀x 6= 0; ±1. Chứng minh. Với n = 0, n = 1 ta có T0 1 2 x + 1 x = 1; T1 1 2 x + 1 x = 1 2 x + 1 x luôn đúng. Giả sử mệnh đề trên đúng đến n. Áp dụng công thức truy hồi của Tn ta có: Tn+1 1 2 x + 1 x = 2. 1 2 x + 1 x Tn 1 2 x + 1 x −Tn−1 1 2 x + 1 x = 1 2 x + 1 x xn + 1 xn − xn−1 + 1 xn−1 = 1 2 xn+1 + 1 xn+1 Ta chứng minh tương tự cho trường hợp của Un (x). Hệ quả 2.1. ∀n ∈ N ta có: Tn (x) = x + √ x2 − 1 n + x − √ x2 − 1 n 2 ; Un (x) = x + √ x2 − 1 n − x − √ x2 − 1 n 2 √ x2 − 1 . Tính chất 2.7. (Tn (x) ; Un (x)) là cặp nghiệm của phương trình Pell đa thức: P2 (x) − x2 − 1 Q2 (x) = 1. Tính chất 2.8. Tn 1 − 2x2 = (−1)n T2n; Un 1 − 2x2 x = (−1)n U2n+1 (x) Tính chất 2.9. Un (x) = xUn−1 (x) + Tn−1 (x) , ∀x ∈ R, n ∈ N∗ . Tính chất 2.10. Tn+1 (x) = xTn (x) − 1 − x2 Un (x) , ∀x ∈ R, n ∈ N. Tính chất 2.11. Tn+m (x)+T|n−m| (x) = 2Tn (x) Tm (x) ; ∀x ∈ R, ∀n, m ∈ N. Tính chất 2.12. Tm (Tn (x)) = Tmn (x) , ∀x ∈ R, ∀m, n ∈ N. Tính chất 2.13. Un (x) = 1 n + 1 T0 n+1 (x). 30
  • 34. Tính chất 2.14. |U (x)| ≤ n ∀x ∈ [−1; 1]; |T0 n (x)| ≤ n2 ∀x ∈ [−1; 1]. Tính chất 2.15. dTn dx = nUn−1; dUn dx = (n + 1) Tn+1 − xUn x2 − 1 ; d2 Tn dx2 = n. nTn − xUn−1 x2 − 1 = n. (n + 1) Tn − Un x2 − 1 . Tổng quát: dp Tn dxp x=±1 = (±1)n+p p−1 Q k=0 n2 − k2 2k + 1 . Nhận xét 2.2. d2 Tn dx2 x=1 = n4 − n2 3 ; d2 Tn dx2 x=−1 = (−1)n n4 − n2 3 . Chứng minh. T00 (1) = lim x→1 n. nTn − xUn−1 x2 − 1 = lim x→1 n. nTn − xUn−1 x − 1 x + 1 = n. lim x→1 nTn − xUn−1 x − 1 lim x→1 (x + 1) = n 2 lim x→1 nTn − xUn−1 x − 1 Áp dụng quy tắc L’Hospital ta có: T00 (1) = n 2 lim x→1 d dx (nTn − xUn−1) d dx (x − 1) = n 2 lim x→1 d dx (nTn − xUn−1) = n 2 lim x→1 n2 Un−1 − Un−1 − x d dx (Un−1) = n 2 n2 Un−1 (1) − Un−1 (1) − lim x→1 d d dx (Un−1) = n4 2 − n2 2 − 1 2 lim x→1 d dx (nUn−1) = n4 2 − n2 2 − T00 n (1) 2 = n4 − n2 3 . Trường hợp còn lại chứng minh tương tự. Tính chất 2.16. 1 − x2 T00 n (x)−xT0 n (x)+n2 Tn (x) = 0, ∀x ∈ R, ∀n ∈ N. Tính chất 2.17. R Undx = Tn+1 n + 1 + C; 31
  • 35. Z Tndx = 1 2 Tn+1 n + 1 − Tn−1 n − 1 + C = nTn+1 n2 − 1 − xTn n − 1 + C. Do dung lượng quy định của luận văn nên tác giả không say sưa vào việc chứng minh các tính chất trên mặc dù việc chứng minh này rất thú vị. Phần lớn các tính chất trên đều được chứng minh theo quy nạp kết hợp các công thức truy hồi. 2.2 Các dạng toán liên quan đến đa thức Cheby- shev Trong các dạng toán về ước lượng đa thức, đa thức Chebyshev đóng vai trò rất quan trọng. Nó vừa là trường hợp xảy ra dấu đẳng thức của nhiều bài toán ước lượng đa thức, vừa là đa thức bổ trợ trong các lời giải và chứng minh.Ta giải quyết một số bài toán liên quan đến đa thức Chebyshev. Bài toán 2.1. Cho đa thức f (x) = 4x3 + (m + 3) x2 + mx. Tìm m ∈ R để |f (x)| ≤ 1, ∀x ∈ [−1; 1] Lời giải. Do f (x) là đa thức bậc 3 và kèm theo điều kiện x ∈ [−1; 1] nên ta có thể liên hệ đến các điểm là các luân điểm của đa thức Chebyshev bậc 3 là xk = cos kπ 3 (k = 0; 1; 2; 3). Cụ thể là x0 = cos 0 = 1; x1 = cos π 3 = 1 2 ; x2 = cos 2π 3 = − 1 2 ; x3 = cos π = −1. Ta có:              |f (1)| ≤ 1 |f (−1)| ≤ 1 f − 1 2 ≤ 1 f 1 2 ≤ 1 ⇒              |2m + 7| ≤ 1 |−1| ≤ 1 − 1 2 + m + 3 4 − m 2 ≤ 1 1 2 + m + 3 4 + m 2 ≤ 1 ⇒            |2m + 7| ≤ 1 1 − m 4 ≤ 1 3m + 5 4 ≤ 1 Suy ra ( −1 ≤ 2m + 7 ≤ 1 −4 ≤ 1 − m ≤ 4 −4 ≤ 3m + 5 ≤ 4 32 Tải bản FULL (77 trang): https://bit.ly/3fQM1u2 Dự phòng: fb.com/KhoTaiLieuAZ
  • 36. Do đó      −4 ≤ m ≤ −3 −3 ≤ m ≤ 5 −3 ≤ m ≤ 1 3 nên m = −3 Đảo lại, khi m = −3 thì f (x) = 4x3 − 3x. Đặt x = cos α với α ∈ [0; π] . Ta có f (x) = cos 3α. Suy ra |f (x)| = |cos 3α| ≤ 1. Bài toán 2.2. Cho đa thức f (x) = 4x3 + ax2 + bx + c. Tìm a, b, c ∈ R để |f (x)| ≤ 1, ∀x ∈ [−1; 1]. Lời giải. Đây là một bài khá hấp dẫn, tôi xin trình bày với nhiều lời giải khác nhau. Cách 1. Bài toán này tương tự với cách ra đề và cách suy luận như bài toán trên. Tuy nhiên, bài toán này có đến 3 tham số đòi hỏi chúng ta phải có những đánh giá sâu sắc hơn. Dựa vào cách suy luận ở bài toán trên, ta để ý đến các luân điểm của đa thức Chebyshev bậc 3 là: x0 = cos 0 = 1; x1 = cos π 3 = 1 2 ; x2 = cos 2π 3 = − 1 2 ; x3 = cos π = −1. Ta có đánh giá sau:              |f (1)| ≤ 1 |f (−1)| ≤ 1 f 1 2 ≤ 1 f − 1 2 ≤ 1 ⇒              |4 + a + b + c| ≤ 1 |−4 + a − b + c| ≤ 1 1 2 + a 4 + b 2 + c ≤ 1 − 1 2 + a 4 − b 2 + c ≤ 1 Suy ra      −5 ≤ a + b + c ≤ −3 (1) 3 ≤ a − b + c ≤ 5 (2) −6 ≤ a + 2b + 4c ≤ 2 (3) −2 ≤ a − 2b + 4c ≤ 6 (4) Sử dụng (1) và (2) ta có: a + b + c ≤ −3 a − b + c ≥ 3 ⇒ b ≤ −3. Sử dụng (3) và (4) ta có: a + 2b + 4c ≥ −6 a − 2b + 4c ≤ 6 ⇒ b ≥ −3. Từ hai đánh giá trên, ta suy ra b = −3. Thay b = −3 vào hệ bất phương trình ở trên, ta được: 33 Tải bản FULL (77 trang): https://bit.ly/3fQM1u2 Dự phòng: fb.com/KhoTaiLieuAZ
  • 37.      −2 ≤ a + c ≤ 0 0 ≤ a + c ≤ 2 −8 ≤ a + 4c ≤ 0 0 ≤ a + 4c ≤ 8 ⇒ a + c = 0 a + 4c = 0 ⇔ a = c = 0. Đảo lại, khi a = 0; b = −3; c = 0 thì f (x) = 4x3 − 3x. Đặt x = cos α với α ∈ [0; π] ta cóf (x) = cos 3α. Suy ra |f (x)| = |cos 3α| ≤ 1. Cách 2. Để cho việc đánh giá và tìm các giá trị của a; b; c được nhanh hơn ta cần quan tâm đến bất đẳng thức quen thuộc |a1 + a2 + ... + an| ≤ |a1| + |a2| + ... + |an| và dấu đẳng thức có được khi các ai cùng dấu. Ta dự đoán đích đến là a = 0; b = −3; c = 0. Quay lại với bài toán. Ta có: |8 + 2b| ≤ |4 + a + b + c| + |4 + b − a − c| ≤ 2. Suy ra |4 + b| ≤ 1. Lại có: |−1 − b| ≤ − 1 2 − 1 4 a − 1 2 b − c + − 1 2 + 1 4 a − 1 2 b + c ≤ 2. Vấn đề ở đây là ta tìm được giá trị của b. Bây giờ nhận thấy 3 ≤ |4 + b| + |−1 − b| ≤ 1 + 2 = 3. Vậy thì dấu bằng xảy ra ở tất cả các bất đẳng thức trên là b = −3; a = c = 0. Đảo lại, khi a = 0; b = −3; c = 0 thì f (x) = 4x3 − 3x. Đặt x = cos α với α ∈ [0; π] ta cóf (x) = cos 3α. Suy ra |f (x)| = |cos 3α| ≤ 1. Cách 3. Do xem xét trên đoạn [−1; 1] nên ta cần quan tâm đến giá trị lớn nhất của hàm số |f (x)| để việc đánh giá dưới đây được thuận lợi. Quay lại bài toán Giả sử tồn tại các số a, b, c thỏa mãn giả thiết bài toán và đặt M = max x∈[−1;1] |f (x)|. Ta có |f (1)| = |4 + a + b + c| ; |f (−1)| = |−4 + a − b + c| ; f 1 2 = 1 2 + a 4 + b 2 + c ; f − 1 2 = − 1 2 + a 4 − b 2 + c . Với cách đặt ở trên, ta suy ra 34 6732069